By sebastian | Wed, Apr 2025
1. A 34-year-old man is brought to the emergency department after being stabbed in the back during an assault. Neurologic exam shows loss of pain and temperature sensation on the left side of the body below the nipple line, loss of vibration and proprioception on the right side below the nipple, and paralysis with hyperreflexia in the right leg. Which of the following is the most likely diagnosis?
(A) Anterior spinal artery syndrome
(B) Central cord (syringomyelia) syndrome
(C) Brown-Séquard syndrome
(D) Amyotrophic lateral sclerosis (ALS)
(E) Tabes dorsalis (neurosyphilis)
Answer and Explanation: The correct answer is (C) Brown-Séquard syndrome. This is a hemisection of the spinal cord (in this case, likely on the right side) leading to ipsilateral loss of dorsal column modalities (vibration/proprioception) below the lesion, ipsilateral upper motor neuron signs (spastic paralysis, hyperreflexia) below the lesion due to corticospinal tract damage, and contralateral loss of pain and temperature below the lesion (spinothalamic tract crosses near the entry level). The combination of these findings is classic for Brown-Séquard. Anterior spinal artery syndrome would cause bilateral loss of motor function and pain/temperature below the lesion, not the asymmetric pattern seen here. Syringomyelia (central cord) typically causes bilateral loss of pain/temperature in a "cape" distribution at the level of the lesion (usually cervical) with possible hand weakness, not the findings in this case. ALS causes combined UMN and LMN signs but is bilateral and does not fit the sensory loss pattern. Tabes dorsalis affects dorsal columns (proprioception) bilaterally and causes ataxia, not the mixed picture here.
2. A 55-year-old man with severe atherosclerosis becomes hypotensive during surgery for an abdominal aortic aneurysm repair. After the surgery, he is unable to move his lower extremities and does not respond to painful stimuli in his legs. Vibration and proprioception sensation in the lower extremities is preserved. Which of the following is the most likely cause of this neurologic deficit?
(A) Occlusion of the anterior spinal artery
(B) Occlusion of a posterior spinal artery
(C) Traumatic hemisection of the spinal cord
(D) Vitamin B12 (cobalamin) deficiency
(E) Multiple sclerosis of the thoracic spinal cord
Answer and Explanation: The correct answer is (A) Occlusion of the anterior spinal artery. Anterior spinal artery syndrome leads to infarction of the anterior two-thirds of the spinal cord. This causes bilateral paralysis (corticospinal tract) and loss of pain and temperature (spinothalamic tract) below the lesion, while dorsal column functions (vibration, proprioception) are spared (since dorsal columns are supplied by posterior spinal arteries). In this scenario, prolonged hypotension likely caused infarction in the territory of the anterior spinal artery. Posterior spinal artery occlusion would primarily affect dorsal columns (loss of proprioception/vibration) instead. Brown-Séquard (hemisection) would cause ipsilateral dorsal column and contralateral spinothalamic loss, not the bilateral pattern here. B12 deficiency causes subacute combined degeneration (dorsal columns and lateral corticospinal tracts), typically presenting with loss of proprioception and spastic paresis, not an isolated anterior cord syndrome. Multiple sclerosis plaques are usually patchy and asymmetric, and unlikely to cause the acute picture described.
3. A 30-year-old woman with a history of a Chiari I malformation presents with bilateral loss of pain and temperature sensation across her shoulders and upper arms (cape-like distribution). Touch, vibration, and proprioception remain intact, and motor function in the upper limbs is mildly impaired. Which of the following is the most likely cause of her symptoms?
(A) Central cord syndrome (syringomyelia)
(B) Brown-Séquard syndrome
(C) Posterior cord syndrome (tabes dorsalis)
(D) Anterior horn cell degeneration (poliomyelitis)
(E) Multiple sclerosis plaque in the thalamus
Answer and Explanation: The correct answer is (A) Central cord syndrome (syringomyelia). A lesion in the central spinal cord, often a syrinx (fluid-filled cavity), typically in the cervical region, can damage the anterior white commissure where the second-order fibers of the spinothalamic tract cross. This causes bilateral loss of pain and temperature in a cape distribution over the shoulders/arms. Light touch and proprioception (dorsal columns) are usually preserved because they run dorsally. Motor neurons in the anterior horn can also be affected, leading to weakness in the upper limbs. Brown-Séquard would produce unilateral deficits and a mix of motor and sensory losses on different sides. Tabes dorsalis (posterior cord) causes loss of proprioception/vibration and sensory ataxia, not selective pain/temperature loss. An anterior horn cell lesion like poliomyelitis would cause flaccid paralysis, not isolated sensory loss. A thalamic MS plaque would cause contralateral sensory deficits of all modalities, not a cape distribution.
4. A 58-year-old man suddenly develops dizziness, vomiting, nystagmus, and difficulty swallowing. On exam, he has a hoarse voice, diminished gag reflex, loss of pain and temperature on the right side of the face, and loss of pain and temperature on the left side of the body. He also has ptosis and miosis on the right side of the face. Which artery is most likely occluded?
(A) Anterior spinal artery
(B) Posterior inferior cerebellar artery (PICA)
(C) Anterior inferior cerebellar artery (AICA)
(D) Basilar artery
(E) Middle cerebral artery (MCA)
Answer and Explanation: The correct answer is (B) Posterior inferior cerebellar artery (PICA). This patient’s signs are classic for Lateral Medullary (Wallenberg) syndrome, which results from infarction in the PICA distribution (lateral medulla). Key features include vestibular symptoms (vertigo, nystagmus from vestibular nucleus damage), difficulty swallowing and hoarseness (nucleus ambiguus, CN IX and X involvement), ipsilateral loss of facial pain and temperature (spinal trigeminal nucleus), contralateral loss of body pain and temperature (spinothalamic tract), and ipsilateral Horner syndrome (descending sympathetic fibers). Occlusion of AICA causes lateral pontine syndrome (which includes facial paralysis due to CN VII involvement and hearing loss due to labyrinthine artery ischemia). Anterior spinal artery occlusion in the medulla (medial medullary syndrome) causes contralateral body paralysis and loss of proprioception with ipsilateral tongue paralysis (CN XII) – very different signs. Basilar artery occlusion tends to cause "locked-in" syndrome if at the base, or other pontine issues if partial. MCA occlusion causes cortical deficits (contralateral face/arm weakness, aphasia if dominant hemisphere, etc.), not brainstem signs.
5. A 45-year-old woman develops sudden facial paralysis on the right side, with inability to close her right eye and drooping of the mouth. She also reports sounds being annoyingly loud in her right ear and some loss of taste from the front of her tongue on that side. There are no other neurological deficits. Which of the following best explains these findings?
(A) Stroke in the left motor cortex (upper motor neuron lesion of CN VII)
(B) Bell palsy (facial nerve CN VII palsy) on the right side
(C) Trigeminal nerve (CN V3) lesion on the right side
(D) Glossopharyngeal nerve lesion on the right side
(E) Neuromuscular junction disorder (myasthenia gravis) affecting facial muscles
Answer and Explanation: The correct answer is (B) Bell palsy (facial nerve CN VII palsy) on the right side. Bell palsy is a peripheral lesion of the facial nerve, often idiopathic, causing ipsilateral paralysis of all muscles of facial expression, including the forehead. This patient cannot close her eye or move her mouth on the right, indicating a lower motor neuron lesion of CN VII. The hyperacusis (sensitivity to loud sounds) is due to paralysis of the stapedius muscle (innervated by CN VII) in the middle ear. Loss of taste from the anterior 2/3 of the tongue is due to involvement of the chorda tympani branch of CN VII. A stroke in the left motor cortex would cause a contralateral lower face paralysis but forehead sparing (upper face receives bilateral UMN innervation), which is not what we see here. Trigeminal V3 lesion would affect jaw muscles and facial sensation, not cause facial droop. Glossopharyngeal nerve lesions affect swallowing, taste in posterior tongue, etc., not full facial paralysis. Myasthenia gravis can cause fluctuating muscle weakness but typically affects ocular muscles first and does not present exactly like an isolated CN VII lesion.
6. A 60-year-old woman presents with double vision, especially when walking downstairs or reading. She tilts her head to the right to compensate for the diplopia. On exam, her left eye drifts upward when she tries to look downward toward her nose. Which nerve is most likely injured?
(A) Right oculomotor nerve (CN III)
(B) Right trochlear nerve (CN IV)
(C) Left trochlear nerve (CN IV)
(D) Left abducens nerve (CN VI)
(E) Right abducens nerve (CN VI)
Answer and Explanation: The correct answer is (C) Left trochlear nerve (CN IV). A trochlear nerve palsy causes weakness of the superior oblique muscle, which primarily intorts and depresses the eye when it is adducted. Patients have difficulty looking down, particularly when reading or descending stairs. They often compensate with a characteristic head tilt away from the affected side (tilting toward the shoulder opposite the lesion) to reduce diplopia. In this case, the patient tilts her head to the right, suggesting a left CN IV palsy (she’s tilting away from the left side because the left eye’s superior oblique is weak). The exam finding of the left eye drifting upward (hypertropia) on attempted down-gaze confirms a left CN IV problem. An oculomotor nerve palsy would cause a "down and out" eye with ptosis and a dilated pupil. An abducens nerve palsy causes inability to abduct the eye (horizontal diplopia, no characteristic head tilt as in CN IV palsy). Right CN IV palsy would cause a left head tilt. So left CN IV is the best match.
7. A 26-year-old man is brought to the ER after a motorcycle collision. He briefly lost consciousness and had a lucid interval at the scene, but then he rapidly deteriorated. A head CT scan shows a lens-shaped, biconvex hyperdense collection that does not cross suture lines. Which vessel was most likely ruptured in this injury?
(A) Bridging veins
(B) Middle meningeal artery
(C) Anterior cerebral artery
(D) Great cerebral vein (of Galen)
(E) Superior sagittal sinus
Answer and Explanation: The correct answer is (B) Middle meningeal artery. The scenario describes an epidural hematoma, which classically occurs after a traumatic head injury with a brief loss of consciousness, a lucid interval, and then rapid decline due to arterial bleeding. On CT, an epidural hematoma appears as a lens-shaped (biconvex) collection of blood that does not cross suture lines because the dura is tightly attached at the sutures. The middle meningeal artery (a branch of the maxillary artery) runs just deep to the pterion region of the skull and is the typical vessel torn by skull fractures in this area. Bridging veins are torn in subdural hematomas, which appear as crescent-shaped collections that can cross sutures and usually have a slower onset (often in elderly or alcoholic patients). Cerebral arteries (like anterior cerebral) cause subarachnoid hemorrhages or intracerebral bleeds, not the described CT pattern. The vein of Galen or dural sinuses would cause different bleeding patterns, not the classic lens shape.
8. An 80-year-old man on anticoagulation therapy for atrial fibrillation falls at home. Over the next two days, he becomes progressively confused and lethargic. CT scan of the head shows a crescent-shaped hyperdense collection of blood that crosses suture lines and covers the convexity of the brain. Which structure is most likely torn in this injury?
(A) Bridging veins
(B) Middle meningeal artery
(C) Inferior sagittal sinus
(D) Vertebral artery
(E) Middle cerebral artery
Answer and Explanation: The correct answer is (A) Bridging veins. This is describing a subdural hematoma, which is common in elderly patients (or those on anticoagulation) after relatively minor head trauma. Subdural hematomas result from tearing of the bridging veins that traverse from the brain surface to the dural venous sinuses. On CT, subdural blood appears as a crescent-shaped (concave) collection that can spread widely and cross suture lines, since the blood accumulates between the dura and arachnoid mater. The onset is often gradual (hours to days) as venous bleeding is slower. In contrast, middle meningeal artery tear causes an epidural hematoma (lens-shaped, acute presentation). Dural sinuses or cerebral arteries would produce different bleeding scenarios (e.g., subarachnoid hemorrhage for cerebral artery aneurysm rupture). Vertebral artery injuries are associated with cervical trauma and cause strokes, not subdural hematomas.
9. A 43-year-old man presents with severe headache, high fever, and double vision. Exam reveals ophthalmoplegia (paralysis of eye movements) of the right eye, ptosis, and loss of corneal reflex on the right. There is also decreased sensation in the right upper face. The patient mentions he had a skin infection on his nose a week ago. Which of the following structures is most likely affected, causing his symptoms?
(A) Cavernous sinus
(B) Superior orbital fissure
(C) Optic canal
(D) Jugular foramen
(E) Internal auditory (acoustic) meatus
Answer and Explanation: The correct answer is (A) Cavernous sinus. The cavernous sinus is a venous sinus at the base of the brain that contains cranial nerves III, IV, V1, V2, and VI, as well as the internal carotid artery. An infection on the face (especially the "danger triangle" of the face, like the nose area) can spread via facial and ophthalmic veins to the cavernous sinus, causing cavernous sinus thrombosis. This leads to ophthalmoplegia (CN III, IV, VI dysfunction – eye movement paralysis and ptosis), loss of the corneal reflex and facial sensory loss in V1 distribution (ophthalmic division of trigeminal), and potentially V2 numbness as well. Ptosis can be due to CN III palsy (levator palpebrae) and sympathetic fiber involvement (Horner syndrome). The superior orbital fissure is an opening that transmits similar nerves (III, IV, V1, VI) into the orbit, but an isolated lesion there (like a fracture) wouldn’t cause a febrile illness as described. The optic canal transmits the optic nerve and ophthalmic artery, not relevant to these motor findings. Jugular foramen syndrome would affect CN IX, X, XI (leading to hoarseness, dysphagia, SCM/trap weakness). Internal acoustic meatus transmits VII and VIII (facial paralysis and hearing loss, which is AICA stroke or acoustic neuroma scenario). The described cluster of findings and cause fits cavernous sinus thrombosis.
10. A 50-year-old man presents with difficulty swallowing and hoarseness that started after he had surgery to remove a tumor at the base of his skull. On exam, he has an absent gag reflex on the right side, palate droop with the uvula deviating to the left, and weakness turning his head to the left (suggesting right sternocleidomastoid weakness). These findings could best be explained by damage to which of the following openings in the skull?
(A) Foramen ovale
(B) Internal acoustic meatus
(C) Hypoglossal canal
(D) Jugular foramen
(E) Superior orbital fissure
Answer and Explanation: The correct answer is (D) Jugular foramen. The jugular foramen transmits cranial nerves IX (glossopharyngeal), X (vagus), and XI (spinal accessory), as well as the jugular vein. A lesion in this area (sometimes called Vernet syndrome when a tumor like a glomus jugulare affects these nerves) will cause dysfunction of all three nerves. Glossopharyngeal (IX) lesion leads to loss of gag reflex (afferent limb) and some taste/sensation loss in the pharynx. Vagus (X) lesion causes palate droop, uvula deviation away from the lesion (toward the normal side), dysphagia, and hoarseness due to vocal cord paralysis. Accessory (XI) lesion causes weakness in the sternocleidomastoid (difficulty turning head to opposite side) and trapezius (shoulder droop) on the affected side. This patient's signs (right gag reflex gone, right palate weakness and SCM weakness) all point to a right jugular foramen lesion affecting IX, X, XI. Foramen ovale transmits V3 (mandibular nerve), which would cause jaw deviation and facial sensory loss, not these findings. Internal acoustic meatus (VII Answer and Explanation (continued): The internal acoustic meatus transmits CN VII and VIII (facial nerve and vestibulocochlear nerve), whose injury would cause facial paralysis and hearing loss, not the palate and SCM findings. The hypoglossal canal transmits CN XII (tongue movements), and the superior orbital fissure transmits CN III, IV, V1, VI (eye movements and sensation). Those do not match the deficits here. Therefore, damage at the jugular foramen best explains this collection of cranial nerve deficits.
11. During a routine check-up, a physician tests a patient's gag reflex. Touching the back of the pharynx on the right side fails to elicit a gag response, but touching the left side of the pharynx produces a normal bilateral gag. Which nerve is most likely impaired in this patient?
(A) Right glossopharyngeal nerve (CN IX)
(B) Right vagus nerve (CN X)
(C) Left glossopharyngeal nerve (CN IX)
(D) Right hypoglossal nerve (CN XII)
(E) Left vagus nerve (CN X)
Answer and Explanation: The correct answer is (A) Right glossopharyngeal nerve (CN IX). The gag reflex has an afferent (sensory) limb mediated primarily by the glossopharyngeal nerve, which senses touch on the pharyngeal mucosa, and an efferent (motor) limb via the vagus nerve causing the gag. In this scenario, touching the right side doesn’t elicit a response (meaning sensation from the right side isn’t getting through), but touching the left side works (so motor pathways and the left sensory limb are intact). This implies a lesion in the right CN IX (the afferent from the right pharynx). If the right vagus (motor) were impaired, touching either side of the pharynx might produce an asymmetric or absent movement on the affected side (like palate not elevating on the right), but here the gag is simply absent only when the right side is stimulated – pointing to the sensory limb on the right. Hypoglossal nerve controls tongue movement, not gag reflex.
12. A 19-year-old man is stabbed with a knife in the left submandibular region. On examination, his tongue deviates to the left when he tries to protrude it. Which nerve was most likely injured?
(A) Left hypoglossal nerve (CN XII)
(B) Right hypoglossal nerve (CN XII)
(C) Left vagus nerve (CN X)
(D) Right vagus nerve (CN X)
(E) Left trigeminal nerve (mandibular division)
Answer and Explanation: The correct answer is (A) Left hypoglossal nerve (CN XII). The hypoglossal nerve innervates the intrinsic and most extrinsic muscles of the tongue (except palatoglossus). A lesion of the hypoglossal nerve causes the tongue to deviate toward the side of the lesion on protrusion because the muscles (genioglossus in particular) on the affected side are weak, and the functional side pushes the tongue toward the weak side. In this case, the tongue deviates to the left, indicating a left-sided CN XII injury. Vagus nerve lesions can cause uvula deviation and swallowing difficulties but do not directly cause tongue deviation (except that bilateral vagus lesions can indirectly affect tongue posture). The mandibular division of trigeminal (V3) innervates jaw muscles, and its injury causes jaw deviation (toward the lesion) but not tongue deviation.
13. A patient is asked to open his jaw. Upon opening, his jaw deviates to the right side. He also has some difficulty chewing and reduced sensation in the lower right face. Which nerve is most likely injured?
(A) Right trigeminal nerve, mandibular division (V3)
(B) Right facial nerve (VII)
(C) Left trigeminal nerve, mandibular division (V3)
(D) Right glossopharyngeal nerve (IX)
(E) Left hypoglossal nerve (XII)
Answer and Explanation: The correct answer is (A) Right trigeminal nerve, mandibular division (V3). The mandibular division of the trigeminal nerve innervates the muscles of mastication (masseter, temporalis, medial and lateral pterygoids) and provides sensory innervation to the lower face/jaw. When the lateral pterygoid muscle (which helps open and protrude the jaw) on one side is weak (due to V3 motor lesion), the jaw deviates toward the side of the lesion upon opening because the intact side’s pterygoid muscle pushes the jaw toward the weak side. In this case, deviation to the right suggests right V3 motor lesion. Additionally, V3 carries sensation from the mandibular region, so sensory loss in the lower right face further confirms a right trigeminal V3 injury. A facial nerve lesion causes facial muscle paralysis, not jaw deviation or lower face numbness. Glossopharyngeal and hypoglossal nerve lesions would not cause jaw deviation.
14. A hypertensive patient performs carotid sinus massage to help slow down his heart rate. The pressure on the carotid sinus is sensed by baroreceptors. Which nerve carries the afferent signals from the carotid sinus to the central nervous system?
(A) Glossopharyngeal nerve (CN IX)
(B) Vagus nerve (CN X)
(C) Phrenic nerve
(D) Sympathetic cervical chain
(E) Trigeminal nerve (CN V)
Answer and Explanation: The correct answer is (A) Glossopharyngeal nerve (CN IX). The carotid sinus (baroreceptor in the carotid artery) and carotid body (chemoreceptor) are innervated by the glossopharyngeal nerve (CN IX) via the carotid sinus nerve (Hering’s nerve). Increased pressure (like with carotid massage) increases afferent firing in CN IX to the medulla, which triggers an increase in parasympathetic outflow via the vagus nerve (efferent) to the heart, thus slowing the heart rate (baroreceptor reflex). The vagus nerve itself carries afferent baroreceptor information from the aortic arch (not the carotid sinus) and is the efferent for both reflexes. The phrenic nerve innervates the diaphragm, unrelated to baroreception. The sympathetic chain carries efferents for fight-or-flight, opposite of what is happening during carotid massage. The trigeminal nerve has nothing to do with visceral baroreception.
15. A 65-year-old man had surgery to remove a large calcified carotid artery plaque. After the procedure, he has difficulty speaking and swallowing. On exam, when he protrudes his tongue, it deviates to the right side. Which nerve was most likely damaged during the surgery?
(A) Right hypoglossal nerve (CN XII)
(B) Left hypoglossal nerve (CN XII)
(C) Right vagus nerve (CN X)
(D) Left glossopharyngeal nerve (CN IX)
(E) Right lingual nerve (branch of V3)
Answer and Explanation: The correct answer is (A) Right hypoglossal nerve (CN XII). The hypoglossal nerve runs near the carotid artery in the neck and can be injured in carotid endarterectomy. An injury to the right CN XII would cause deviation of the tongue to the right on protrusion (the tongue deviates toward the side of a peripheral hypoglossal lesion due to unopposed action of the normal side’s genioglossus muscle). This fits the exam finding. Difficulty with speech (dysarthria) and swallowing can also occur if the tongue is weak. A vagus nerve lesion could cause some swallowing difficulty and hoarseness, but tongue deviation specifically indicates the hypoglossal nerve. Glossopharyngeal lesion would affect gag reflex and taste from posterior tongue but not tongue movement. The lingual nerve (branch of V3) carries sensation from the anterior tongue; its injury causes sensory loss, not deviation.
16. A 29-year-old woman requires a lumbar puncture for suspicion of meningitis. The physician locates the iliac crest as a landmark. At which vertebral level should the needle be inserted for a safe lumbar puncture in an adult, and which structure will the needle pass through just before entering the subarachnoid space?
(A) Between L3/L4; ligamentum flavum
(B) Between L1/L2; posterior longitudinal ligament
(C) Between T12/L1; dura mater only (no ligaments at this level)
(D) Between L5/S1; anterior longitudinal ligament
(E) Between C7/T1; ligamentum nuchae
Answer and Explanation: The correct answer is (A) Between L3/L4; ligamentum flavum. In adults, the spinal cord ends at approximately the L1–L2 vertebral level, but the subarachnoid space (containing the cauda equina) continues to about S2. To safely perform a lumbar puncture without damaging the cord, the needle is usually inserted at the L3/L4 or L4/L5 interspace (the L4 level is approximated by the line connecting the iliac crests, known as Tuffier’s line). The needle will pass through several structures: skin, fascia, supraspinous ligament, interspinous ligament, ligamentum flavum (a thick elastic ligament between vertebral laminae, often yielding a characteristic "pop"), then through the dura mater and arachnoid into the subarachnoid space where CSF is withdrawn. The posterior longitudinal ligament is anterior to the spinal cord (inside the spinal canal against the vertebral bodies) and not pierced in a lumbar puncture. The anterior longitudinal ligament is on the front of the vertebral bodies (completely inaccessible from the back). C7/T1 is too high (spinal cord present, and that level is not for LP). Ligamentum nuchae is the cervical continuation of the supraspinous ligament, not relevant to lumbar region.
17. A 58-year-old male with a history of untreated syphilis presents with difficulty walking. He has a wide-based gait and frequently watches his feet while walking. On exam, he has positive Romberg sign (loses balance when eyes are closed). He also has lightning-like shooting pains in his legs and urinary incontinence. Which part of the nervous system is most likely damaged?
(A) Dorsal columns of the spinal cord
(B) Anterior horn cells in lumbar cord
(C) Lateral corticospinal tracts
(D) Spinothalamic tracts
(E) Cerebellar vermis
Answer and Explanation: The correct answer is (A) Dorsal columns of the spinal cord. This patient’s presentation is classic for tabes dorsalis, a manifestation of tertiary syphilis that involves degeneration of the dorsal columns and dorsal roots (especially in lumbosacral region). The dorsal columns (gracile and cuneate tracts) carry proprioception, vibration, and fine touch. Damage leads to sensory ataxia – difficulty with coordination and balance that is worse in the dark or with eyes closed (positive Romberg sign) because visual compensation is removed. Lightning pains (shooting, stabbing pains in the legs) are due to dorsal root involvement. Incontinence can occur from loss of sensory input. The lateral corticospinal tracts (motor) are not primarily affected in tabes dorsalis (those are involved in B12 deficiency, for example). Spinothalamic tracts (pain/temperature) are typically intact in tabes dorsalis. Anterior horn cells are affected in diseases like poliomyelitis or ALS (LMN signs). Cerebellar vermis damage causes truncal ataxia, but Romberg is specifically a test of proprioceptive (dorsal column) dysfunction, not cerebellar function.
18. When a light is shone into a patient’s right eye, neither pupil constricts. However, when light is shone into the left eye, both pupils constrict normally. Which structure is most likely damaged in this patient?
(A) Right optic nerve (CN II)
(B) Right oculomotor nerve (CN III)
(C) Left optic nerve (CN II)
(D) Left oculomotor nerve (CN III)
(E) Right abducens nerve (CN VI)
Answer and Explanation: The correct answer is (A) Right optic nerve (CN II). In the pupillary light reflex, the optic nerve carries afferent light input to the brain, and the oculomotor nerve (parasympathetic fibers from the Edinger-Westphal nucleus traveling with CN III) carries the efferent signal to constrict the pupils. In this scenario, shining light in the right eye yields no pupillary constriction in either eye, indicating that the afferent limb on the right is not functioning (the brain never "sees" the light). However, shining light in the left eye causes both pupils to constrict, indicating that both efferent pathways (CN III on each side) are intact and that the left afferent is intact. Therefore, the lesion must be in the right optic nerve. A right oculomotor lesion would cause an efferent defect: shining light in either eye would cause left pupil constriction but right pupil would stay dilated. A left optic lesion would result in the opposite pattern of the one described. Abducens nerve controls lateral rectus muscle, unrelated to the light reflex.
19. A 40-year-old man is tested for hearing loss. A tuning fork placed on the middle of his forehead localizes sound to the left ear. When the tuning fork is placed on the mastoid behind the left ear until he can't hear it and then moved next to the left ear canal, he still cannot hear it. On the right side, he hears the sound longer in air than on bone. What is the most likely diagnosis?
(A) Left conductive hearing loss
(B) Left sensorineural hearing loss
(C) Right conductive hearing loss
(D) Right sensorineural hearing loss
(E) Normal hearing bilaterally
Answer and Explanation: The correct answer is (A) Left conductive hearing loss. This is the classic interpretation of the Weber and Rinne tests. The Weber test (tuning fork on forehead) localizes to the left ear, meaning either a conductive loss in the left ear or a sensorineural loss in the right ear. The Rinne test on the left shows bone conduction greater than air conduction (BC > AC) on that side, which is abnormal and indicates a conductive loss in the left ear (in conductive loss, bone conduction is heard longer or louder than air conduction in the affected ear). On the right side, Rinne is normal (air > bone). This combination (Weber lateralizes to left AND left Rinne is abnormal with BC > AC) confirms a left conductive hearing loss. In sensorineural loss on one side, Weber would lateralize to the opposite (better) ear and Rinne in the affected ear would still show AC > BC (because Rinne only tells you if there's a conductive component). Here, left is clearly conductive. Causes could be otitis media, otosclerosis, or cerumen in the left ear.
20. A 36-year-old man presents with shooting pains radiating from his lower back down the lateral aspect of his right leg into the dorsum of his foot and big toe. On exam, he has weakness in dorsiflexing his right foot (foot drop) and decreased sensation between the first and second toes. An MRI shows an intervertebral disc herniation. Which nerve root is most likely compressed by this herniated disc?
(A) L3
(B) L4
(C) L5
(D) S1
(E) S2
Answer and Explanation: The correct answer is (C) L5. The patient’s symptoms indicate sciatica involving the L5 nerve root. A posterolateral disc herniation at the L4-L5 level typically compresses the traversing L5 nerve root. L5 radiculopathy causes pain radiating down the lateral leg into the dorsum of the foot and the big toe. Motor weakness often involves dorsiflexion (tibialis anterior, extensor hallucis longus), leading to a foot drop, and patients may have trouble walking on their heels. Sensory loss can occur on the dorsum of the foot, especially between the great toe and second toe (the L5 dermatome). In contrast, an S1 radiculopathy (often from L5-S1 disc herniation) causes pain down the back of the leg into the lateral foot, weakness in plantarflexion (difficulty tiptoeing), and loss of the ankle reflex. L4 radiculopathy (from L3-L4 disc) would affect the anterior leg, knee, and medial foot, with a reduced patellar reflex. L3 or L2 issues would involve anterior thigh, not lower leg and foot.
21. A 45-year-old woman undergoes a thyroidectomy for a large thyroid tumor. After surgery, she has a hoarse voice. Laryngoscopy reveals that her right vocal cord is paralyzed (does not move). Which nerve was most likely injured during the thyroidectomy?
(A) Right external branch of the superior laryngeal nerve
(B) Right recurrent laryngeal nerve
(C) Right glossopharyngeal nerve
(D) Right hypoglossal nerve
(E) Right spinal accessory nerve
Answer and Explanation: The correct answer is (B) Right recurrent laryngeal nerve. The recurrent laryngeal nerves (branches of the vagus, X) innervate all the intrinsic muscles of the larynx except the cricothyroid. These muscles move the vocal cords. In thyroid surgery, especially if removing the inferior thyroid poles, the recurrent laryngeal nerve (which runs just behind the thyroid gland, in the tracheoesophageal groove) is at risk of injury. A unilateral injury causes hoarseness and a fixed vocal cord on that side. The external branch of the superior laryngeal nerve innervates only the cricothyroid muscle; injury to it can cause difficulty with high pitch (and slight hoarseness), but not a paralyzed cord on exam. Glossopharyngeal nerve is related to pharynx sensation and gag, not laryngeal muscles. Hypoglossal nerve moves the tongue. Accessory nerve moves SCM and trapezius (injury causes shoulder droop, not voice changes).
22. A 30-year-old man accidentally swallows a large fish bone, which becomes lodged in his piriform recess (a region in the laryngopharynx just lateral to the laryngeal inlet). Shortly afterward, he notices that his cough reflex when food or saliva goes down "the wrong pipe" is diminished. Which nerve is most likely injured by the sharp bone?
(A) Glossopharyngeal nerve (CN IX) – pharyngeal branch
(B) Internal laryngeal nerve (branch of superior laryngeal, from CN X)
(C) Recurrent laryngeal nerve (from CN X)
(D) Hypoglossal nerve (CN XII)
(E) Lingual nerve (branch of V3)
Answer and Explanation: The correct answer is (B) Internal laryngeal nerve (branch of superior laryngeal, from CN X). The piriform recess (piriform fossa) is in the laryngopharynx on either side of the laryngeal orifice. The internal branch of the superior laryngeal nerve (from the vagus nerve) runs just deep to the mucosa in this area and provides sensory innervation to the larynx above the vocal folds. A fish bone lodged here can damage this nerve, leading to loss of sensation from the upper larynx. This would diminish the afferent limb of the cough reflex (which normally prevents aspiration). Glossopharyngeal nerve provides sensation to the oropharynx and tongue (gag reflex afferent), not the larynx. The recurrent laryngeal nerve provides sensation below the vocal cords and motor to most laryngeal muscles; an injury there usually causes vocal cord paralysis and loss of sensation below the cords (affecting the cough reflex for material in the trachea). Hypoglossal nerve moves the tongue, and lingual nerve provides sensation to anterior tongue – neither is related to the cough reflex.
23. A 50-year-old woman undergoes removal of lymph nodes in the posterior triangle of the neck for melanoma. After surgery, she notices that her right shoulder is drooping and she has difficulty elevating her right arm above horizontal and turning her head to the left against resistance. Which nerve was most likely damaged?
(A) Right spinal accessory nerve (CN XI)
(B) Right dorsal scapular nerve
(C) Right long thoracic nerve
(D) Right suprascapular nerve
(E) Right axillary nerve
Answer and Explanation: The correct answer is (A) Right spinal accessory nerve (CN XI). The spinal accessory nerve innervates the trapezius and sternocleidactomastoid (SCM) muscles. In the posterior triangle of the neck (bounded by SCM, trapezius, and clavicle), CN XI travels superficially and is susceptible to injury during lymph node dissections or trauma. Damage to the accessory nerve causes shoulder droop (due to trapezius paralysis) and difficulty elevating or abducting the arm above the horizontal position (because trapezius is needed to rotate the scapula upward). It also causes weakness in turning the head to the opposite side (SCM function). These match the patient’s deficits. The dorsal scapular nerve innervates the rhomboids (retract scapula) and levator scapulae; injury causes difficulty retracting the scapula, not the described deficits. Long thoracic nerve injury causes winged scapula (serratus anterior palsy) but not inability to shrug. Suprascapular nerve innervates supraspinatus/infraspinatus (abduction initiation and external rotation); injury causes weak initiation of abduction and external rotation, but not a drooping shoulder. Axillary nerve innervates deltoid and teres minor; injury (like in surgical neck fractures) causes inability to abduct arm beyond 15° and numbness over lateral shoulder, but trapezius and SCM would be intact.
24. A 60-year-old man with a long history of smoking presents with drooping of his right upper eyelid, constricted right pupil, and dry skin on the right side of his face (lack of sweating). A Pancoast tumor at the lung apex is suspected. The patient’s syndrome results from compression of which of the following?
(A) Cervical sympathetic chain (leading to Horner syndrome)
(B) Oculomotor nerve (CN III)
(C) Facial nerve (CN VII)
(D) Trigeminal nerve (CN V)
(E) Trochlear nerve (CN IV)
Answer and Explanation: The correct answer is (A) Cervical sympathetic chain. The combination of ptosis (drooping eyelid), miosis (pupil constriction), and anhidrosis (lack of sweating) on one side of the face defines Horner syndrome, which results from disruption of the sympathetic pathways to the face. A common cause is a Pancoast tumor (superior sulcus tumor) in the apex of the lung, which can compress the cervical sympathetic chain or stellate ganglion as it travels to the head. Oculomotor nerve palsy can cause ptosis, but it leads to a dilated pupil (due to parasympathetic loss) rather than a constricted one, and does not cause anhidrosis of the face. Facial nerve lesions cause facial muscle paralysis and possibly loss of lacrimation or salivation, but not the triad described. Trigeminal nerve lesions cause facial numbness and weakness in mastication if motor division involved, not ptosis/miosis. Trochlear nerve lesions cause an eye movement deficit (superior oblique palsy) but none of the features of Horner’s.
25. A 6-year-old boy suffers frequent nosebleeds (epistaxis) from Kiesselbach’s plexus area in his anterior nasal septum. Which of the following arteries is a major contributor to the blood supply of this area?
(A) Sphenopalatine artery (branch of maxillary artery)
(B) Posterior ethmoidal artery (branch of ophthalmic artery)
(C) Superior labial artery (branch of facial artery)
(D) Greater palatine artery (branch of maxillary artery)
(E) All of the above
Answer and Explanation: The correct answer is actually all of the above, but since that’s not a given single choice, the best choice from those listed is (A) Sphenopalatine artery. Kiesselbach’s plexus (Little’s area) in the anteroinferior nasal septum is a region where several arterial supplies anastomose. The key contributors include the sphenopalatine artery (from the maxillary artery), the anterior ethmoidal artery (from the ophthalmic artery), the greater palatine artery (from maxillary artery, via incisive canal), and the septal branch of the superior labial artery (from the facial artery)【9†L838-L846】. Frequent nosebleeds in children often originate here. Of the options given, the sphenopalatine artery is often considered the major blood supply to the nasal septum and a significant contributor to severe bleeds. (If “all of the above” were an option, it would encompass Kiesselbach’s plexus contributors; in practice, epistaxis can involve any or all of these vessels).
26. A 4-year-old girl is brought to the clinic with a midline neck mass. The mass moves upward when she sticks out her tongue or when she swallows. It has been slowly enlarging over time. What is the most likely diagnosis?
(A) Thyroglossal duct cyst
(B) Second branchial cleft cyst
(C) Cystic hygroma (lymphangioma)
(D) Dermoid cyst
(E) Reactive lymph node
Answer and Explanation: The correct answer is (A) Thyroglossal duct cyst. A thyroglossal duct cyst arises from a persistent thyroglossal duct – a remnant of the tract that the thyroid gland takes during embryological descent from the foramen cecum at the tongue base to the neck. These cysts are typically midline (near the hyoid bone) and characteristically move with swallowing or tongue protrusion because they are connected to the tongue via tissues at the foramen cecum. A second branchial cleft cyst, in contrast, is usually lateral (along the anterior border of the sternocleidomastoid) and does not move with swallowing. A cystic hygroma is a congenital lymphangioma often in the posterior triangle (also lateral). A dermoid cyst can occur midline but usually lower and doesn’t move with swallowing like a thyroglossal cyst. Reactive lymph nodes are usually lateral (along chains) unless it’s submental, but those wouldn’t move with tongue protrusion either.
27. A newborn baby has a unilateral cleft lip. Which of the following developmental processes failed to occur properly?
(A) Fusion of the maxillary prominence with the medial nasal prominence
(B) Fusion of the lateral palatine shelves (palatal processes)
(C) Fusion of the mandibular prominences
(D) Descent of the palatine shelves due to a small mandible
(E) Fusion of the lateral nasal prominence with the maxillary prominence
Answer and Explanation: The correct answer is (A) Fusion of the maxillary prominence with the medial nasal prominence. A cleft lip results from the failure of the maxillary prominence to fuse with the medial nasal prominence (which is part of the intermaxillary segment) during embryonic development. This fusion normally forms the upper lip (philtrum area); if it fails on one side, a unilateral cleft lip occurs. Cleft palate, on the other hand, involves failure of the lateral palatine processes (shelves) to fuse with each other and with the nasal septum (that would be option B, but that causes cleft palate, not cleft lip). The mandibular prominences failing to fuse would cause a cleft in the lower jaw (which is very rare). Option D describes Pierre Robin sequence (small mandible leading to cleft palate), not cleft lip. Lateral nasal prominence fusion is involved in alar (side) nose formation, not upper lip.
28. A patient’s right eye is unable to look downwards when it is turned inward (adducted). Which extraocular muscle is most likely paralyzed?
(A) Superior oblique
(B) Inferior oblique
(C) Superior rectus
(D) Inferior rectus
(E) Medial rectus
Answer and Explanation: The correct answer is (A) Superior oblique. The superior oblique muscle, innervated by the trochlear nerve (CN IV), is responsible for depressing the eye, especially when the eye is adducted (turned inward), and also intorts the eye. If the right eye cannot move down when looking medially (for example, difficulty reading or going down stairs), it indicates a problem with the right superior oblique muscle (or its nerve). The inferior oblique elevates the eye when adducted. The superior rectus elevates the eye (and intorts when adducted), and the inferior rectus depresses the eye best when it is abducted (turned outward). The medial rectus adducts the eye horizontally. Given the movement described (inability to depress an adducted eye), superior oblique is the muscle that fits that action.
29. A 25-year-old woman had an infection of her palatine tonsil that required a tonsillectomy. After the surgery, she noticed loss of taste and general sensation on the posterior third of her tongue and an absence of the gag reflex on that side. Which nerve was most likely injured during the tonsillectomy?
(A) Glossopharyngeal nerve (CN IX)
(B) Vagus nerve (CN X)
(C) Hypoglossal nerve (CN XII)
(D) Lingual nerve (branch of CN V3)
(E) Chorda tympani (branch of CN VII)
Answer and Explanation: The correct answer is (A) Glossopharyngeal nerve (CN IX). The glossopharyngeal nerve runs in the tonsillar bed and provides sensory innervation to the oropharynx and palatine tonsil area, including general sensation and taste from the posterior one-third of the tongue. It is also the afferent limb of the gag reflex. Injury to CN IX during a tonsillectomy can lead to loss of sensation and taste in the back of the tongue on that side and a diminished gag reflex (when touching that side of the pharynx). The vagus nerve provides motor to the pharynx (gag reflex efferent) and sensation to the laryngopharynx, but taste in the posterior tongue is CN IX. Hypoglossal nerve moves the tongue muscles, not taste or gag sensation. The lingual nerve (V3) carries general sensation from the anterior 2/3 of tongue, and chorda tympani (VII) carries taste from anterior 2/3 – neither innervates the posterior tongue or pharynx.
30. After a surgical procedure in the infratemporal fossa, a patient experiences a markedly dry mouth and difficulty with parotid gland secretion. It is suspected that the parasympathetic fibers to the parotid gland were damaged. The preganglionic parasympathetic fibers destined for the parotid travel in which cranial nerve?
(A) Glossopharyngeal nerve (CN IX)
(B) Facial nerve (CN VII)
(C) Trigeminal nerve (mandibular division, V3)
(D) Vagus nerve (CN X)
(E) Hypoglossal nerve (CN XII)
Answer and Explanation: The correct answer is (A) Glossopharyngeal nerve (CN IX). Parasympathetic innervation of the parotid gland originates with the glossopharyngeal nerve. Specifically, the preganglionic fibers leave CN IX as the lesser petrosal nerve (after synapsing in the otic ganglion) and then hitchhike on the auriculotemporal nerve (a branch of V3) to reach the parotid gland. So CN IX carries the preganglionic parasympathetics that eventually cause the parotid to secrete. The facial nerve (CN VII) carries parasympathetics to the submandibular and sublingual glands via chorda tympani (to submandibular ganglion), and to lacrimal/nasal glands via the greater petrosal nerve (pterygopalatine ganglion). The trigeminal (V3) does carry the auriculotemporal nerve which distributes the postganglionic fibers to the parotid, but trigeminal itself has no autonomic fibers origin (it’s just a carrier). Vagus innervates viscera, not the parotid. Hypoglossal has no parasympathetic function at all.
31. During a neurological exam, a physician tests the corneal reflex in a patient. When the right cornea is touched with a cotton wisp, neither eye blinks. When the left cornea is touched, both eyes blink. Which nerve is most likely impaired?
(A) Right ophthalmic branch of trigeminal (CN V1)
(B) Right facial nerve (CN VII)
(C) Left ophthalmic branch of trigeminal (CN V1)
(D) Left facial nerve (CN VII)
(E) Right oculomotor nerve (CN III)
Answer and Explanation: The correct answer is (A) Right ophthalmic branch of trigeminal (CN V1). The corneal reflex involves afferent signals via the ophthalmic division of the trigeminal nerve (V1) sensing the cornea being touched, and efferent signals via the facial nerve (CN VII) causing both orbicularis oculi muscles to blink. In this scenario, touching the right cornea yields no response, but touching the left cornea yields a normal blink in both eyes. This means the efferent pathway (CN VII to both eyes) is intact, and the problem is in sensing touch on the right cornea – a lesion in the right CN V1 (specifically the nasociliary branch that innervates the cornea). A right facial nerve lesion would prevent blinking on the right side (so when left cornea touched, only left eye would blink). A left V1 lesion would cause the opposite pattern. CN III is unrelated to the corneal reflex (it moves the eye and controls pupil constriction).
32. A 35-year-old man has chronic sinusitis affecting his maxillary sinus. The physician wants to improve drainage of the maxillary sinus by creating an opening into the nasal cavity. Into which part of the nasal cavity does the maxillary sinus normally drain?
(A) Superior meatus
(B) Middle meatus
(C) Inferior meatus
(D) Sphenoethmoidal recess
(E) Nasopharynx
Answer and Explanation: The correct answer is (B) Middle meatus. The maxillary sinus (as well as the frontal sinus and anterior ethmoid air cells) drains into the middle nasal meatus, specifically into the semilunar hiatus region of the middle meatus. The superior meatus receives drainage from the posterior ethmoid air cells. The sphenoidal sinus drains into the sphenoethmoidal recess (which is above the superior concha). The nasolacrimal duct drains into the inferior meatus (this is why crying leads to a runny nose, but no sinus drains there except that duct). The nasopharynx is not a location for sinus drainage (it is where the eustachian tube opens). Therefore, to facilitate drainage of the maxillary sinus, one would target the middle meatus.
33. An infection from the upper lip and nose area (so-called "danger triangle" of the face) can spread to the cavernous sinus, leading to cavernous sinus thrombosis. This spread is possible because of venous connections. Which vein is the most direct route for such an infection to spread from the face to the cavernous sinus?
(A) Ophthalmic veins (superior or inferior)
(B) Facial vein via the angular vein
(C) Pterygoid plexus veins
(D) External jugular vein
(E) Retromandibular vein
Answer and Explanation: The correct answer is (A) Ophthalmic veins. The so-called "danger triangle" of the face (the area around the nose and upper lip) has venous drainage through the facial vein, which has connections (via the ophthalmic veins, particularly the superior ophthalmic vein through the orbit) to the cavernous sinus. Infections (like a furuncle or picking at a pimple in that region) can spread via the facial vein to the ophthalmic veins and then directly to the cavernous sinus, since these veins are valveless. While the facial vein is the source, the superior ophthalmic vein is the direct channel into the cavernous sinus. The pterygoid plexus communicates with the cavernous sinus as well, but the more classic route from the upper lip/nose is through the ophthalmic vein. The external jugular and retromandibular veins drain the face/scalp to the subclavian and internal jugular systems and are not directly connected to the cavernous sinus.
34. A patient with Bell’s palsy (facial nerve lesion) complains that sounds are very loud in his affected ear. This hyperacusis (sensitivity to loud sounds) is due to paralysis of which muscle?
(A) Stapedius
(B) Tensor tympani
(C) Levator veli palatini
(D) Tensor veli palatini
(E) Stylopharyngeus
Answer and Explanation: The correct answer is (A) Stapedius. The stapedius muscle in the middle ear attaches to the stapes (the stirrup bone) and functions to dampen the vibration of the stapes in response to loud sounds, thereby protecting the inner ear. The stapedius is innervated by the facial nerve (CN VII). In Bell’s palsy (an idiopathic peripheral facial nerve palsy), paralysis of stapedius can cause hyperacusis – ordinary sounds seem uncomfortably loud because the stapes is not being stabilized. Tensor tympani is another middle ear muscle (innervated by V3) that dampens vibration of the tympanic membrane via the malleus; its paralysis (which is rare) could also cause some sound issues, but Bell’s palsy doesn’t affect V3. Levator and tensor veli palatini are soft palate muscles (elevating palate, opening eustachian tube) – not related to sound dampening. Stylopharyngeus (innervated by CN IX) elevates the pharynx, unrelated to hearing.
35. An elderly man with a history of poor dental hygiene presents with a foul-smelling chronic sinus infection. Imaging reveals a communication between a maxillary molar tooth root and the maxillary sinus. Which sinus is most likely infected due to this odontogenic source?
(A) Maxillary sinus
(B) Frontal sinus
(C) Ethmoid air cells
(D) Sphenoid sinus
(E) Mastoid air cells
Answer and Explanation: The correct answer is (A) Maxillary sinus. The roots of the upper maxillary teeth (especially premolars and first molars) lie in close proximity to the floor of the maxillary sinus. Infections of these teeth or dental procedures can sometimes create a fistula or allow bacteria to spread into the maxillary sinus, causing a sinusitis. This is known as odontogenic sinusitis and is most common in the maxillary sinus because of that anatomic relationship. The frontal, ethmoid, and sphenoid sinuses are not directly related to the teeth. Mastoid air cells communicate with the middle ear, not related to teeth at all.
36. A 31-year-old woman, who had rheumatic fever as a child, presents with shortness of breath and fatigue. On exam, she has a holosystolic (pansystolic) murmur best heard at the apex of the heart and radiating to the left axilla. Echocardiography shows regurgitation of one of her heart valves. Where is the best place on the chest wall to auscultate this murmur?
(A) Right lower border of the sternum (4th intercostal space)
(B) Left 2nd intercostal space, parasternal (left upper sternal border)
(C) Right 2nd intercostal space, parasternal (right upper sternal border)
(D) Left 5th intercostal space, midclavicular line
(E) Left 2nd intercostal space, midclavicular line
Answer and Explanation: The correct answer is (D) Left 5th intercostal space, midclavicular line. This location is the cardiac apex (near the point of maximal impulse) and is the best spot to hear murmurs of the mitral valve. A holosystolic murmur radiating to the axilla is characteristic of mitral regurgitation. In this case, the patient likely has mitral regurgitation (maybe due to rheumatic heart disease damaging the valve). The apex beat area (5th ICS at midclavicular line on the left) is where the mitral valve sounds are heard loudest【9†L859-L867】【9†L869-L872】. The right lower sternal border is the tricuspid area (4th/5th ICS parasternal) for tricuspid murmurs or VSD holosystolic murmurs. Left 2nd ICS parasternal is the pulmonic area (for pulmonic murmurs or splitting of S2). Right 2nd ICS parasternal is the aortic area (aortic stenosis, etc.). Left 2nd ICS midclavicular line is not a standard auscultation site (there is no valve there; perhaps it was meant to confuse with axilla radiation).
37. A 70-year-old man reports episodes of chest pain and fainting on exertion. On exam, he has a harsh crescendo-decrescendo systolic murmur best heard at the base of the heart in the right second intercostal space, with radiation to the neck. Which cardiac valve is most likely affected?
(A) Aortic valve
(B) Pulmonic valve
(C) Mitral valve
(D) Tricuspid valve
(E) Patent ductus arteriosus (not a valve, but a vessel)
Answer and Explanation: The correct answer is (A) Aortic valve. A harsh systolic ejection murmur at the right upper sternal border (2nd ICS, right side) that radiates to the carotids (neck) is classic for aortic stenosis. The patient’s symptoms of chest pain (angina) and syncope with exertion, along with the murmur, strongly suggest aortic stenosis, often due to age-related calcification of the aortic valve. The pulmonic area is at the left 2nd ICS; a pulmonic stenosis murmur is heard there but radiates to left shoulder and is much less common in older adults. Mitral and tricuspid murmurs are heard at the apex and left lower sternal border respectively, and typically don’t radiate to the neck. A PDA causes a continuous murmur (machine-like) best heard at left infraclavicular area, not fitting this description.
38. A 68-year-old man with a history of myocardial infarction presents with episodes of dizziness and near-fainting. His ECG shows third-degree (complete) atrioventricular block, with dissociation between P waves and QRS complexes. The infarct likely affected the blood supply of the atrioventricular (AV) node. In most individuals, which artery supplies the AV node?
(A) Right coronary artery (RCA)
(B) Left anterior descending artery (LAD)
(C) Left circumflex artery (LCX)
(D) Acute marginal artery
(E) Left coronary artery (LCA) trunk
Answer and Explanation: The correct answer is (A) Right coronary artery (RCA). In about 85% of people who are right dominant, the AV node is supplied by the AV nodal branch which usually comes off the right coronary artery (near where RCA gives off the posterior descending artery). An infarction in the RCA (particularly an inferior MI) can therefore cause ischemia of the AV node, leading to AV block (as seen in this patient). In left-dominant circulation (about 8-10% of people), the AV nodal artery comes from the left circumflex. LAD supplies the anterior wall and septum including the His bundle and bundle branches, but not typically the AV node (though LAD infarct can cause bundle branch blocks). The acute marginal is a branch of RCA that supplies the right ventricle wall (not specifically AV node). The left coronary artery trunk splits into LAD and LCX; it doesn’t directly give an AV nodal branch.
39. A 55-year-old man has chest pain and is found to have an anterior wall myocardial infarction with ST elevations in leads V2-V4 on ECG. Which coronary artery is most likely occluded?
(A) Left anterior descending (LAD) artery
(B) Right coronary artery (RCA)
(C) Left circumflex artery (LCX)
(D) Posterior descending artery (PDA)
(E) Obtuse marginal branch of LCX
Answer and Explanation: The correct answer is (A) Left anterior descending (LAD) artery. ST elevation in leads V2-V4 indicates an anteroseptal wall MI, which corresponds to the territory of the LAD (also known as the anterior interventricular artery). The LAD supplies the anterior portion of the left ventricle, the anterior 2/3 of the interventricular septum, and the apex. Occlusion of the LAD is the most common cause of an MI and would cause the pattern described. The RCA typically corresponds to inferior leads (II, III, aVF) if occluded. The LCX corresponds to lateral leads (I, aVL, V5, V6) if occluded (especially in left-dominant systems it can also cause inferior changes). The PDA is a branch usually of RCA (in right-dominant) or LCX (left-dominant) and supplies inferior/posterior septum; an isolated PDA occlusion might cause posterior MI changes (often V7-V9 or tall R in V1-V2). Obtuse marginal is a branch of LCX supplying lateral wall (leads V5, V6). So anteroseptal is LAD.
40. A 60-year-old man with a history of lung cancer presents with swelling of his face, neck, and arms, and prominent veins in these areas. He also reports headaches and feeling of fullness in the head, worse when bending over. You suspect superior vena cava (SVC) syndrome. Which of the following is the most likely cause of these findings?
(A) Compression of the superior vena cava by an apical lung tumor
(B) Thrombosis of the external jugular veins
(C) Occlusion of the carotid arteries
(D) Heart failure causing jugular venous distension
(E) Blockage of the thoracic duct
Answer and Explanation: The correct answer is (A) Compression of the superior vena cava by an apical lung tumor. Superior vena cava syndrome is usually caused by an obstruction of the SVC, often due to a mediastinal mass such as a Pancoast tumor (apical lung tumor) or other lung cancers/lymphomas. This leads to impaired venous drainage from the head, neck, and upper extremities, causing swelling and venous distension in those areas, along with headaches and fullness due to increased intracranial pressure. External jugular vein thrombosis alone would not cause bilateral arm swelling and all those symptoms because collateral drainage exists. Carotid occlusion would cause stroke, not venous congestion. Heart failure can cause bilateral jugular venous distension and edema, but usually also leg edema; and not typically the visible venous collaterals in chest. Thoracic duct blockage causes lymphedema (often chylothorax or unilateral arm edema if at subclavian vein junction), not this constellation.
41. An alcoholic man is brought to the hospital after being found unconscious. It is suspected he aspirated vomit while lying flat on his back. Aspiration pneumonia is most likely to develop in which part of his lungs?
(A) Right superior lobe – apical (upper) segment
(B) Right middle lobe
(C) Right inferior lobe – superior segment
(D) Left inferior lobe – superior segment
(E) Left superior lobe – lingula segment
Answer and Explanation: The correct answer is (C) Right inferior lobe – superior segment. When a person aspirates while supine (lying on their back), the aspirated material tends to go to the superior segment of the right inferior lobe. This is because the right main bronchus is more vertical and wider, and when supine, the posterior segments of the upper lobes and superior segment of lower lobe are gravity-dependent. In upright aspiration (like choking while sitting up), it often ends up in the right inferior lobe’s basal segments (particularly the right lower lobe). But specifically, for supine patients, the right superior segment of the lower lobe is a common site. The right middle lobe or left lingula might get aspirate if lying on the right side or inclined, but supine favors the superior segment of right lower lobe. The left side is less common due to a more angled left bronchus.
42. A physician needs to perform a thoracentesis (pleural fluid tap) in a patient with a pleural effusion. He decides to insert the needle in the midaxillary line on the right side. To avoid injuring the lung, he should insert the needle at which of the following intercostal spaces and positions?
(A) Above the upper border of the 9th rib in the midaxillary line
(B) Below the lower border of the 5th rib in the midclavicular line
(C) Above the upper border of the 5th rib at the midclavicular line
(D) Below the lower border of the 9th rib in the midaxillary line
(E) At the level of the 2nd intercostal space in the midclavicular line
Answer and Explanation: The correct answer is (A) Above the upper border of the 9th rib in the midaxillary line. For a thoracentesis, the needle is typically placed in the 8th, 9th, or 10th intercostal space on the midaxillary line to access the costodiaphragmatic recess (where fluid accumulates). On the right side, one common site is the 9th intercostal space at the midaxillary line. Importantly, the needle should be inserted just above the superior border of the rib to avoid the intercostal neurovascular bundle, which runs along the inferior border of each rib. If the needle is placed below the 9th rib (choice D), you risk hitting the diaphragm or abdominal organs on the right (like liver) and also injuring the neurovascular bundle of 9th rib. The midclavicular line approach (5th ICS midclavicular) is often used for chest tube insertion for pneumothorax (2nd ICS) or pericardiocentesis (left 5th ICS near sternum), but not for pleural fluid at the base. Always remember: go above the rib, not below.
43. In a cricothyrotomy (emergency cricothyroidotomy), a tube is inserted through a membrane to establish an airway. Which structure is incised in this procedure?
(A) Cricothyroid membrane
(B) Thyrohyoid membrane
(C) Tracheal rings below the thyroid isthmus
(D) Median thyrohyoid ligament
(E) Cricoid cartilage
Answer and Explanation: The correct answer is (A) Cricothyroid membrane. An emergency airway (cricothyrotomy) is established by making an incision through the skin and the cricothyroid membrane (also called the cricothyroid ligament) which lies between the thyroid cartilage and the cricoid cartilage in the midline. This provides a quick access to the airway below a blocked upper airway. The thyrohyoid membrane is higher up (between the hyoid bone and thyroid cartilage) and is not used in emergency airway (a high airway would be a surgical airway called a thyrotomy, not common). Tracheostomy (not cricothyrotomy) involves opening the trachea, often between rings or below the thyroid isthmus, but in acute situations cricothyroid membrane is preferred because it’s superficial and avascular. The cricoid cartilage itself is a solid ring and is not cut; the goal is the membrane above it.
44. A 45-year-old man comes to the ER with severe right upper quadrant pain due to acute cholecystitis (gallbladder inflammation). He also experiences pain in his right shoulder. The referred shoulder pain is due to irritation of the diaphragm. Which nerve carries pain from the diaphragmatic pleura and peritoneum, leading to referred pain in the shoulder?
(A) Phrenic nerve
(B) Vagus nerve
(C) Greater splanchnic nerve
(D) Intercostal nerves (T7-T11)
(E) Suprascapular nerve
Answer and Explanation: The correct answer is (A) Phrenic nerve. The phrenic nerve (C3, C4, C5) innervates the diaphragm (both motor and sensory to central parts). Pain from diaphragmatic pleura or diaphragmatic peritoneum (like from a gallbladder irritation contacting the diaphragm or subdiaphragmatic air, etc.) is referred to the shoulder region (C4 dermatome), because the phrenic nerve shares spinal segments with the supraclavicular nerves. This is why gallbladder diaphragmatic irritation can cause right shoulder pain (Kehr’s sign is left shoulder pain from splenic rupture/diaphragm irritation, same mechanism via phrenic). The vagus nerve carries parasympathetic information and visceral reflexes, not somatic pain. Lower intercostal nerves innervate peripheral diaphragm and abdominal wall (pain referred to local area, not shoulder). The greater splanchnic nerve (T5-T9) carries visceral pain from foregut, which can cause epigastric pain, but the shoulder pain specifically is phrenic. Suprascapular nerve innervates shoulder muscles, not related to referred pain path.
45. A 29-year-old woman is undergoing surgery for a neck tumor, during which the thoracic duct is accidentally damaged. Lymphatic fluid begins to accumulate. Where does the thoracic duct normally drain its lymph into the venous circulation?
(A) Junction of the left internal jugular and left subclavian veins
(B) Junction of the right internal jugular and right subclavian veins
(C) Directly into the superior vena cava
(D) Azygous vein in the chest
(E) Left brachiocephalic vein, at its origin from subclavian and jugular
Answer and Explanation: The correct answer is (A) Junction of the left internal jugular and left subclavian veins. The thoracic duct is the main lymphatic channel draining lymph from the entire body except the right head/neck, right upper limb, and right thorax (those drain via the right lymphatic duct). The thoracic duct ascends through the thorax and empties into the venous system at the left venous angle, which is the junction of the left internal jugular vein and left subclavian vein (which then form the left brachiocephalic vein). Damage to the thoracic duct can cause leakage of lymph (chyle) into the chest (chylothorax). The right lymphatic duct drains at the right internal jugular-subclavian junction. The thoracic duct does not drain into the SVC or azygos directly. (Note: the left brachiocephalic vein is formed by the union of the left subclavian and left internal jugular – essentially the same location as answer A, but the classic description is the junction of those two veins).
46. A transesophageal echocardiogram (TEE) is performed by inserting an ultrasound probe into the esophagus behind the heart. Which cardiac chamber is immediately anterior to the esophagus and therefore best visualized in this view?
(A) Left atrium
(B) Right atrium
(C) Left ventricle
(D) Right ventricle
(E) Coronary sinus
Answer and Explanation: The correct answer is (A) Left atrium. The left atrium is located at the posterior aspect of the heart, immediately anterior to the esophagus. This relationship is so close that an enlarged left atrium (like in mitral stenosis) can cause dysphagia by compressing the esophagus. In a transesophageal echocardiogram (where the probe is in the esophagus behind the heart), the structures best seen are the left atrium, the interatrial septum, and other posterior structures like the aorta. The right atrium is more anterior and to the right side. The left ventricle is anterolateral to left atrium. The right ventricle forms most of the anterior surface of the heart (farthest from the esophagus). The coronary sinus runs in the atrioventricular groove posteriorly, but the chamber that directly abuts the esophagus is the left atrium.
47. During a surgical procedure, a clamp is placed on the inferior vena cava as it passes through the diaphragm. At what vertebral level does the IVC pass through the diaphragm, and what other structure accompanies it at that opening?
(A) T8 level; the phrenic nerve (right phrenic nerve)
(B) T10 level; the vagus nerves
(C) T12 level; the azygos vein
(D) T8 level; the vagus nerves
(E) T10 level; the thoracic duct
Answer and Explanation: The correct answer is (A) T8 level; the phrenic nerve (right phrenic nerve). The major structures passing through the diaphragm and their vertebral levels can be remembered by the mnemonic "I 8 10 Eggs At 12": IVC at T8, Esophagus at T10, Aorta at T12. Specifically, the IVC (inferior vena cava) goes through the central tendon of the diaphragm at the T8 level and is accompanied by the right phrenic nerve (since the IVC is on the right, the right phrenic goes with it). The esophagus passes at T10 with the vagal trunks (CN X) and esophageal branches of left gastric vessels. The aorta passes behind the diaphragm at T12 along with the thoracic duct and sometimes the azygos vein (though the azygos actually arcs and joins SVC above the diaphragm; it passes through or behind the crura around T12). So, IVC at T8 with right phrenic is correct.
48. A 20-year-old man is found to have high blood pressure in his arms, low blood pressure in his legs, and weak femoral pulses. Chest X-ray shows notching of the ribs. Which of the following is the most likely diagnosis?
(A) Post-ductal coarctation of the aorta
(B) Patent ductus arteriosus
(C) Takayasu arteritis (aortic arch syndrome)
(D) Thoracic outlet syndrome
(E) Atrial septal defect (ASD)
Answer and Explanation: The correct answer is (A) Post-ductal coarctation of the aorta. This condition is a narrowing of the aorta typically just distal to the insertion of the ductus arteriosus (ligamentum arteriosum) – hence "post-ductal." It leads to hypertension in the upper extremities (because the blood has difficulty passing the narrowing, so pressure proximal to it is high) and relative hypotension in the lower extremities (blood flow beyond the coarctation is reduced). Patients often develop collateral circulation via intercostal arteries to bypass the coarctation, which causes enlarged intercostal arteries that erode the undersides of the ribs, leading to rib notching on X-ray. Femoral pulses are delayed or weak compared to brachial (radio-femoral delay). Patent ductus arteriosus causes a continuous murmur and doesn’t typically cause differential limb BP. Takayasu arteritis (pulseless disease) affects aortic arch branches and can cause arm pulse discrepancies, but not rib notching classically, and it’s rare and in young women typically. Thoracic outlet syndrome compresses subclavian vessels or brachial plexus, affecting an arm, not systemic BP difference upper vs lower. ASD causes abnormal heart sounds but no blood pressure difference in limbs.
49. A child inhales a small peanut. Which of the following locations is the peanut most likely to lodge?
(A) Right main bronchus
(B) Left main bronchus
(C) Left upper lobe bronchus
(D) Right upper lobe bronchus
(E) Carina of the trachea
Answer and Explanation: The correct answer is (A) Right main bronchus. The right main (primary) bronchus is more vertical, wider, and shorter than the left. Therefore, inhaled foreign bodies are more likely to go down the right side. Often, they end up in the right lower lobe bronchus (particularly the superior segment of the right lower lobe if the aspiration occurred supine, or a basal segment if upright), but the first place it usually lodges is the right main bronchus or one of its branches. The left main bronchus is angled more acutely (more horizontal) due to the heart, and it’s narrower, so foreign objects less commonly go there. The right upper lobe bronchus branches off the right main bronchus at a very steep angle near the carina, so large objects usually continue to the middle or lower lobe bronchus rather than going up into the right upper lobe. The carina is a ridge and foreign bodies usually go one way or the other; lodging right at the carina is possible but less likely than going into the right bronchial tree.
50. A 66-year-old woman with long-standing hypertension is found to have an enlarged left atrium on an echocardiogram due to mitral stenosis. Which of the following symptoms might occur as a result of left atrial enlargement?
(A) Difficulty swallowing (dysphagia) for solids
(B) Hoarseness of voice
(C) Left-sided Horner syndrome
(D) Hemoptysis (coughing up blood)
(E) All of the above
Answer and Explanation: The best answer from the choices is (A) Difficulty swallowing (dysphagia) for solids, though in fact multiple choices could occur. An enlarged left atrium can compress the esophagus (which lies directly posterior to the left atrium) leading to dysphagia, particularly for solids. It can also impinge on the left recurrent laryngeal nerve (which hooks under the aortic arch adjacent to the left atrium) leading to a hoarse voice (Ortner syndrome). It can cause compression of the left main bronchus or pulmonary veins leading to congestion and hemoptysis. However, of the listed options, dysphagia is a classic direct result of LA enlargement. Hoarseness (choice B) is also a known effect (due to recurrent laryngeal nerve compression), and hemoptysis (D) can occur due to rupture of bronchial veins from elevated pressure. Horner syndrome (C) is not typically from LA enlargement; Horner’s is more related to sympathetic chain compression (e.g., apical lung tumor). If “all of the above” were an option including hemoptysis, that might be considered, but since it’s not clearly correct (Horner doesn’t fit), dysphagia is the hallmark symptom.
51. A 6-week-old male infant is brought to the clinic by his parents due to a swelling in his groin and scrotum, noticed when he cries. On exam, a bulge extends into the right scrotum. The mass is reducible and goes away when the child is calm. The hernia sac is felt lateral to the inferior epigastric vessels. Which type of hernia is most likely?
(A) Indirect inguinal hernia
(B) Direct inguinal hernia
(C) Femoral hernia
(D) Umbilical hernia
(E) Hydrocele (not a hernia, but fluid)
Answer and Explanation: The correct answer is (A) Indirect inguinal hernia. An indirect inguinal hernia is common in infants and children due to a persistent processus vaginalis (a congenital patent connection between the peritoneal cavity and scrotum). It goes through the deep inguinal ring, lateral to the inferior epigastric vessels, into the inguinal canal, and can extend into the scrotum. It is covered by all three layers of spermatic fascia. The scenario describes exactly that (bulge into scrotum, present when crying/increased intra-abdominal pressure). A direct inguinal hernia usually occurs in older men, protrudes through Hesselbach’s triangle medial to inferior epigastrics, and rarely goes into the scrotum. A femoral hernia appears as a bulge below the inguinal ligament in the upper thigh/groin area (more common in women). An umbilical hernia would be at the navel. A hydrocele is fluid in the tunica vaginalis which can also cause scrotal swelling, but it’s not described as reducible with crying; plus, the description of the hernia sac lateral to vessels is classic for indirect hernia.
52. A 58-year-old obese woman notices a bulge in her right groin. On exam, the bulge is located below the inguinal ligament and just medial to the femoral vein. It becomes more prominent when she coughs. Which type of hernia is most likely?
(A) Femoral hernia
(B) Indirect inguinal hernia
(C) Direct inguinal hernia
(D) Spigelian hernia
(E) Incisional hernia
Answer and Explanation: The correct answer is (A) Femoral hernia. A femoral hernia protrudes through the femoral canal, which is below the inguinal ligament, in the femoral triangle area. It emerges through the femoral ring into the upper thigh/groin. The key relationships: in the femoral sheath, from lateral to medial, the structures are Nerve (femoral nerve) – Artery – Vein – Empty space (femoral canal) – Lymphatics (mnemonic NAVEL). A femoral hernia will present as a bulge in the upper thigh or groin, medial to the femoral vein (since it comes through the femoral canal). This type is more common in women and has a higher risk of incarceration/strangulation. An indirect inguinal hernia is above the inguinal ligament and lateral to epigastrics, going towards scrotum (in men). A direct inguinal is above the ligament, medial to epigastrics, often in older men. Spigelian hernia is along the semilunar line (lateral edge of rectus abdominis). Incisional hernia occurs at the site of a surgical incision.
53. A tumor is located at the splenic flexure of the colon (left colic flexure). This region is a watershed area between arterial supplies. The primary artery supplying this part of the colon is which of the following?
(A) Inferior mesenteric artery (IMA)
(B) Superior mesenteric artery (SMA)
(C) Celiac trunk
(D) Middle colic artery
(E) Inferior epigastric artery
Answer and Explanation: The correct answer is (A) Inferior mesenteric artery (IMA). The splenic flexure (left colic flexure) is the junction between the transverse colon and the descending colon. It is a watershed area between the SMA and IMA circulations. The IMA typically supplies the structures from the distal transverse colon (including the splenic flexure) to the upper rectum. Specifically, the left colic branch of the IMA supplies the area of the splenic flexure. The SMA supplies the colon up to approximately the proximal 2/3 of the transverse colon. The celiac trunk supplies foregut (up to proximal duodenum) and not the colon at all. The middle colic artery is a branch of the SMA that supplies the proximal/mid transverse colon, but by the time we reach splenic flexure, blood is primarily from the left colic (IMA). Inferior epigastric is a branch of external iliac and supplies abdominal wall, unrelated to colon.
54. A 22-year-old man comes to the ER with diffuse abdominal pain that started around his umbilicus and later moved to the right lower quadrant. He now has severe pain at McBurney’s point (two-thirds of the way from the umbilicus to the anterior superior iliac spine) with rebound tenderness. He is febrile. Which of the following best explains the initial referred pain near the umbilicus?
(A) Visceral afferent pain fibers from the appendix enter the spinal cord at T10
(B) Somatic pain fibers from the parietal peritoneum of the appendix refer to T4
(C) The phrenic nerve refers pain to the umbilicus
(D) Inflammation of the colon causes pain referred to mid-abdomen
(E) The appendix is supplied by the vagus nerve which causes periumbilical pain
Answer and Explanation: The correct answer is (A) Visceral afferent pain fibers from the appendix enter the spinal cord at T10. In early appendicitis, the visceral peritoneum of the appendix is irritated, producing a dull, diffuse pain around the periumbilical area. This is because the afferent visceral pain fibers from the midgut (which includes the appendix) travel via sympathetic nerves to reach the T10 spinal segment, and T10 dermatomal region corresponds to the umbilicus. As the inflammation progresses and the parietal peritoneum in the right lower quadrant becomes involved, the pain localizes to the RLQ (somatic pain via spinal nerves). So initial referred pain at the umbilicus is due to visceral pain referred to the T10 dermatome. Parietal peritoneum of appendix would produce localized RLQ pain (somatic) once irritated. The phrenic nerve refers pain to shoulder (C4), not umbilicus. Vagus carries parasympathetic, not pain; visceral pain travels with sympathetics usually. T4 is nipple line dermatome (not related here).
55. A 60-year-old man has a history of liver cirrhosis and portal hypertension. He presents with hemorrhoidal bleeding. On exam, he has internal hemorrhoids. Which veins are dilated in internal hemorrhoids, and into which vein do they ultimately drain?
(A) Superior rectal veins; drain into the inferior mesenteric vein (portal system)
(B) Inferior rectal veins; drain into the internal pudendal vein (caval system)
(C) Middle rectal veins; drain into the internal iliac vein
(D) External rectal plexus; drains into inferior vena cava
(E) Left colic vein; drains into inferior mesenteric vein
Answer and Explanation: The correct answer is (A) Superior rectal veins; drain into the inferior mesenteric vein (portal system). Internal hemorrhoids occur above the pectinate line and are due to dilation of the internal rectal venous plexus. The superior rectal veins (which drain the upper rectum and anal canal above pectinate line) are part of the portal circulation, draining into the inferior mesenteric vein (and then splenic vein -> portal vein). In portal hypertension, these can engorge, leading to internal hemorrhoids. Internal hemorrhoids are typically not as painful (visceral innervation). External hemorrhoids (below pectinate line) involve the inferior rectal veins which drain into the internal pudendal -> internal iliac -> IVC (systemic circulation), and are very painful due to somatic innervation (pudendal nerve). Middle rectal veins (to internal iliac) form minor connections between the two systems. In this patient with cirrhosis, the portal side (superior rectal via IMV) is congested, causing internal hemorrhoids.
56. During a femoral arterial catheterization in the groin, the physician accidentally punctures a structure immediately lateral to the femoral artery within the femoral triangle. The patient subsequently has weakness of knee extension and numbness of the anterior thigh and medial leg. Which structure was likely damaged?
(A) Femoral nerve
(B) Femoral vein
(C) Lateral femoral cutaneous nerve
(D) Saphenous nerve
(E) Obturator nerve
Answer and Explanation: The correct answer is (A) Femoral nerve. In the femoral triangle, from lateral to medial, the arrangement is NAVEL: Nerve, Artery, Vein, (Empty space), Lymphatics. The femoral nerve lies just lateral to the femoral artery. If a catheter or needle goes too far lateral while trying to enter the femoral artery, it can injure the femoral nerve. Damage to the femoral nerve would cause weakness or inability to extend the knee (quadriceps paralysis) and loss of sensation to the anterior thigh and medial leg (the medial leg via the saphenous nerve, which is the terminal sensory branch of the femoral nerve). The femoral vein is medial to the artery, not lateral. Lateral femoral cutaneous nerve is more lateral (near ASIS) and purely sensory to lateral thigh (injury causes meralgia paresthetica, not knee extension weakness). The saphenous nerve is a branch of femoral nerve in the thigh (pure sensory to medial leg); injuring just it would cause sensory loss in medial leg but not the motor loss described. The obturator nerve is in the medial thigh (adductors), not near the femoral artery.
57. Which of the following structures is retroperitoneal?
(A) Descending colon
(B) Stomach
(C) Spleen
(D) Jejunum
(E) Liver
Answer and Explanation: The correct answer is (A) Descending colon. Retroperitoneal structures are those that lie behind the peritoneum (either primarily or secondarily). A useful mnemonic is SAD PUCKER: Suprarenal (adrenal) glands, Aorta/IVC, Duodenum (2nd-4th parts), Pancreas (head, neck, body), Ureters, Colon (ascending and descending parts), Kidneys, Esophagus (thoracic part), Rectum (partially). The descending colon (as well as the ascending colon) is secondarily retroperitoneal (it started intraperitoneal in development but became fused to the posterior wall). In contrast, the stomach and spleen are intraperitoneal (with mesenteries). The jejunum is intraperitoneal (suspended by the mesentery proper). The liver is intraperitoneal (though it’s attached to the diaphragm by ligaments, most of it is peritoneal except bare area). So descending colon is the retroperitoneal one in the list.
58. A 50-year-old man with alcoholic cirrhosis presents vomiting blood. Endoscopy shows bleeding esophageal varices. These varices result from a portosystemic anastomosis between which veins?
(A) Left gastric vein and esophageal veins (azygos system)
(B) Splenic vein and left renal vein
(C) Superior mesenteric vein and inferior vena cava
(D) Inferior mesenteric vein and lumbar veins
(E) Hepatic veins and portal vein
Answer and Explanation: The correct answer is (A) Left gastric vein and esophageal veins (azygos system). Esophageal varices are a result of portal hypertension causing backup of blood at the anastomosis between the portal circulation and systemic circulation in the lower esophagus. The portal component is the left gastric vein (which drains into the portal vein via the splenic vein) and the systemic component is the esophageal veins that drain into the azygos vein (and then SVC). In portal hypertension, the left gastric vein is engorged and the blood finds alternate pathways through esophageal submucosal veins, causing varices. Splenic to left renal is a shunt surgeons create (Warren shunt) but not a natural variceal route. Superior mesenteric to IVC doesn’t directly anastomose; SMV drains to portal. Inferior mesenteric to lumbar is not a major site (IMV drains to splenic). Hepatic veins drain liver to IVC (post-liver), not part of portal systemic connection in terms of varices.
59. A 27-year-old man is stabbed in the left posterior thorax just below the 10th rib in the midscapular line. He is hypotensive and the ER physician suspects injury to an organ in that area. Which organ is most likely lacerated from this injury?
(A) Spleen
(B) Left kidney
(C) Stomach
(D) Pancreas
(E) Left lung
Answer and Explanation: The correct answer is (A) Spleen. The spleen is located in the left upper quadrant of the abdomen, tucked under the rib cage. It roughly spans ribs 9–11 on the left side at the midaxillary to posterior axillary line. A stab wound below the left 10th rib in a posterior lateral position is very likely to hit the spleen, which is the most commonly injured organ in abdominal trauma, especially blunt trauma. The left kidney is more posterior and lower (around T12-L2 vertebral levels, protected by 11th and 12th ribs more medially). The stomach is more anterior and to the left, not usually injured from a posterior stab at that location. The pancreas is deep (retroperitoneal) and crosses midline; a wound that lateral probably misses it. The left lung is above the diaphragm inside the thorax; below the 10th rib is below the diaphragm (and midscapular line is lateral to the likely pleural reflection, which ends about rib 10 at midax line, but midscapular is a bit more posterior—still, likely below pleura at 10th rib). So spleen is the best fit.
60. During a hysterectomy, the surgeon is ligating the uterine arteries. The resident is cautioned to be careful to avoid injury to a structure that runs immediately under (inferior to) the uterine artery in the cardinal (transverse cervical) ligament. Which structure is this?
(A) Ureter
(B) Ovarian artery
(C) Round ligament of uterus
(D) Uterine (Fallopian) tube
(E) Internal iliac vein
Answer and Explanation: The correct answer is (A) Ureter. A classic anatomical relationship: “Water under the bridge.” In females, the uterine artery (from internal iliac) passes over the ureter (the “water” being urine in the ureter, and the “bridge” being the uterine artery) near the lateral fornix of the vagina. So during a hysterectomy, when the uterine artery is tied off, care must be taken not to accidentally ligate or cut the ureter, which is just inferior to it. The ovarian vessels (in the suspensory ligament of ovary) also cross the ureter but at the pelvic brim (the ovarian artery is higher up and not typically an issue in a routine hysterectomy unless removing ovaries). The round ligament of the uterus goes into the inguinal canal and isn’t near the uterine artery. The fallopian tube is in the upper edge of the broad ligament, not in the cardinal ligament. The internal iliac vein is posterior and not directly under uterine artery at that site like the ureter is.
61. A 28-year-old woman presents with sudden onset of severe right lower quadrant pain. Ultrasound suggests a torsion (twisting) of the right ovary, compromising its blood supply. The ovary’s blood supply travels in which ligament that would need to be untwisted during surgery?
(A) Suspensory ligament of the ovary (infundibulopelvic ligament)
(B) Ovarian ligament
(C) Round ligament of the uterus
(D) Broad ligament (mesovarium portion)
(E) Uterosacral ligament
Answer and Explanation: The correct answer is (A) Suspensory ligament of the ovary (infundibulopelvic ligament). The suspensory ligament of the ovary (also called the infundibulopelvic ligament) attaches the ovary to the lateral pelvic wall and contains the ovarian vessels (ovarian artery and vein). In ovarian torsion, the ovary twists around this ligament, cutting off its blood supply and causing acute pain. Surgical correction involves untwisting this and possibly fixing it. The ovarian ligament (proper ovarian ligament) connects the ovary to the uterus (no major vessels, just a remnant of gubernaculum). The round ligament of the uterus goes from the uterus through the inguinal canal to labia majora (contains no major vessels, just small artery of Sampson). The broad ligament is a fold of peritoneum supporting the uterus, tubes, ovaries (the mesovarium part attaches to ovary) but the major ovarian vessels are specifically in the suspensory ligament. Uterosacral ligaments support the uterus to the sacrum, no ovarian vessels there.
62. A woman in labor is having a difficult time with pain. The obstetrician decides to perform a pudendal nerve block for pain relief during a forceps delivery. The anesthetic should be injected near which bony landmark as a guide?
(A) Ischial spine
(B) Ischial tuberosity
(C) Anterior superior iliac spine
(D) Sacral promontory
(E) Pubic symphysis
Answer and Explanation: The correct answer is (A) Ischial spine. A pudendal nerve block is often done by injecting local anesthetic near the pudendal nerve as it crosses the lateral aspect of the sacrospinous ligament, which attaches to the ischial spine. Transvaginal palpation of the ischial spine is a common approach, and then anesthetic is infiltrated around that area to block the pudendal nerve (S2-S4), which carries sensation from the perineum and genital area. The ischial tuberosity is lower (what we sit on); a block near there is less direct. ASIS is used for lateral femoral cutaneous nerve block sometimes, not pudendal. Sacral promontory is too high/deep (landmark in pelvis for OB measurements). Pubic symphysis is midline anterior, not relevant for pudendal block.
63. A 24-year-old man suffers a pelvic fracture in a motorcycle accident. He subsequently has difficulty achieving an erection. The urologist suspects damage to the autonomic nerves that promote penile erection. Which nerves are responsible for erection?
(A) Pelvic splanchnic nerves (S2-S4)
(B) Pudendal nerves
(C) Sacral sympathetic chain (sacral splanchnics)
(D) Hypogastric nerves (lumbar sympathetics)
(E) Genitofemoral nerves
Answer and Explanation: The correct answer is (A) Pelvic splanchnic nerves (S2-S4). Penile erection is primarily mediated by parasympathetic innervation (point = parasympathetic, as in the saying “Point and Shoot” – point for erection, shoot for ejaculation via sympathetic). The pelvic splanchnic nerves (S2-S4) carry preganglionic parasympathetic fibers that innervate the erectile tissues (via the inferior hypogastric plexus) causing vasodilation and engorgement (erection). Damage to these (for example, in pelvic fracture or prostate surgery) can cause erectile dysfunction. The pudendal nerve (S2-S4 somatic) carries sensation from the penis and motor to perineal muscles; it’s involved in the reflexes and contraction of bulbospongiosus during ejaculation, but the initiation of erection is parasympathetic. Sacral splanchnics are sympathetic (they contribute to sympathetic innervation in pelvis), which are involved in ejaculation (emission phase) and detumescence (they cause vasoconstriction after). Hypogastric nerves (from lumbar symp) carry sympathetic to pelvic plexus, also mainly involved in emission (sperm and fluid movement during ejaculation). Genitofemoral is an L1-L2 nerve mostly for cremaster reflex and sensation to upper scrotum, not erection.
64. During a difficult delivery, a midline episiotomy (surgical cut) is performed to enlarge the vaginal opening. The incision is made through the perineal body. Which of the following structures attaches to the perineal body and might be affected?
(A) Bulbospongiosus muscle
(B) Levator ani muscle
(C) Ischiocavernosus muscle
(D) Obturator internus muscle
(E) Piriformis muscle
Answer and Explanation: The correct answer is (A) Bulbospongiosus muscle. The perineal body is a fibromuscular mass in the midline of the perineum between the vagina (or bulb of penis in males) and the anus. It serves as an attachment point for several perineal muscles including the bulbospongiosus, superficial transverse perineal, deep transverse perineal, and fibers from the external anal sphincter and levator ani. In a midline episiotomy, the perineal body is cut, which can affect these muscles. Of the options given, bulbospongiosus (which helps in clitoral/penile erection and supports pelvic floor) attaches there. Levator ani (pelvic floor muscle) also has some connection to the perineal body, but primarily attaches to fascia and around anorectal junction – it could be indirectly affected if the perineal body is severely damaged. Ischiocavernosus attaches to ischial tuberosities and crus of clitoris/penis (not the perineal body). Obturator internus and piriformis are pelvic wall muscles, not related to perineal body.
65. A 72-year-old man presents with painless jaundice and weight loss. Imaging reveals a mass in the head of the pancreas compressing the common bile duct. Which of the following best describes the anatomical location of the pancreas head and the consequence of its enlargement?
(A) The pancreas head is nestled in the curve of the duodenum; its enlargement can obstruct the common bile duct leading to jaundice
(B) The pancreas head lies left of the spleen; its enlargement compresses the splenic artery causing gastric varices
(C) The pancreas head lies behind the stomach; its enlargement compresses the portal vein causing ascites
(D) The pancreas head is anterior to the transverse colon; its enlargement causes bowel obstruction
(E) The pancreas head is in the splenorenal ligament; its enlargement causes left kidney compression
Answer and Explanation: The correct answer is (A) The pancreas head is nestled in the curve of the duodenum; its enlargement can obstruct the common bile duct leading to jaundice. The pancreas head sits in the C-loop of the duodenum (the duodenum wraps around it). The common bile duct passes through or very near the head of the pancreas to empty into the duodenum (at the ampulla of Vater). A tumor in the head of the pancreas can compress the distal bile duct, causing backup of bile and painless obstructive jaundice (often with a palpable gallbladder – Courvoisier’s sign). The pancreas tail is in the splenorenal ligament near the spleen, but a head tumor wouldn’t affect splenic artery or cause gastric varices (portal hypertension scenario). Compression of the portal vein could occur with a large pancreatic tumor but ascites usually comes from cirrhosis/portal HTN, and portal vein compression could cause varices more than ascites; that scenario is less common than bile duct obstruction. Pancreas is mostly retroperitoneal (except tail), and not anterior to transverse colon in a way to cause obstruction (transverse colon is anterior/inferior to pancreas body). Pancreas head isn’t near the kidney (tail is near left kidney, but head is near duodenum/right side).
66. A 2-year-old boy passes a red, currant jelly-like stool. He has been fussy and having abdominal pain. A Meckel diverticulum is suspected. A Meckel diverticulum is a remnant of what embryonic structure?
(A) Vitelline (omphalomesenteric) duct
(B) Allantois
(C) Processus vaginalis
(D) Midgut dorsal mesentery
(E) Urachus
Answer and Explanation: The correct answer is (A) Vitelline (omphalomesenteric) duct. A Meckel diverticulum is a true diverticulum (containing all layers of the small intestine wall) usually located in the ileum, about 2 feet from the ileocecal valve. It results from a persistence of the vitelline duct (omphalomesenteric duct) that normally connects the midgut to the yolk sac in early embryonic life. When it doesn’t obliterate completely, it can form a diverticulum that sometimes contains ectopic gastric or pancreatic tissue, leading to bleeding (currant jelly stool) or inflammation mimicking appendicitis. The allantois is an embryologic structure whose remnant is the urachus; persistence can cause a urachal fistula at the bellybutton (patent urachus) with urine leakage. Processus vaginalis is a peritoneal extension in the scrotum; persistence can cause indirect inguinal hernia or hydrocele. Dorsal mesentery forms the mesentery of gut, not directly related to a diverticulum formation. Urachus (from allantois) issues cause midline bladder-umbilical connection problems, not GI bleeding.
67. A newborn vomits green (bilious) fluid and does not tolerate feeding. Imaging reveals a constriction of the second part of the duodenum with the pancreatic tissue encircling it. Which embryologic anomaly is the most likely cause?
(A) Annular pancreas due to abnormal migration of the ventral pancreatic bud
(B) Hypertrophic pyloric stenosis due to muscular overgrowth
(C) Duodenal atresia due to failure of recanalization
(D) Malrotation of the midgut with volvulus
(E) Pancreatic divisum due to failure of duct fusion
Answer and Explanation: The correct answer is (A) Annular pancreas due to abnormal migration of the ventral pancreatic bud. An annular pancreas occurs when the ventral pancreatic bud, which normally rotates around to fuse with the dorsal bud, splits or migrates improperly to encircle the duodenum. This forms a ring of pancreatic tissue around the second part of the duodenum, leading to narrowing and obstruction. Newborns with annular pancreas often have bilious vomiting because the obstruction is usually distal to the ampulla of Vater (so bile from the liver can still enter proximal duodenum, making vomit green). Hypertrophic pyloric stenosis causes projectile non-bilious vomiting (obstruction proximal to bile entry) and usually presents ~3 weeks after birth. Duodenal atresia (associated with Down syndrome) is due to failure to recanalize and causes bilious vomiting, but imaging typically shows a “double-bubble” and there’s no pancreatic tissue ring, just an atretic segment. Malrotation with volvulus can cause obstruction and bilious vomiting but the question specifically mentioned pancreatic tissue encircling duodenum. Pancreatic divisum is usually asymptomatic, related to pancreatic duct anatomy, not duodenal obstruction.
68. A 35-year-old man presents with left-sided flank pain and blood in his urine. He also notes that his left scrotum has a “bag of worms” appearance (varicocele). An abdominal CT scan shows compression of his left renal vein. What is the most likely cause of these findings?
(A) Compression of the left renal vein between the superior mesenteric artery and aorta (Nutcracker syndrome)
(B) Left kidney stone lodged at the ureteropelvic junction
(C) Aneurysm of the abdominal aorta compressing the IVC
(D) Thrombosis of the left testicular vein
(E) Left adrenal tumor invading the renal vein
Answer and Explanation: The correct answer is (A) Compression of the left renal vein between the superior mesenteric artery and aorta (Nutcracker syndrome). This scenario describes Nutcracker syndrome, where the left renal vein is compressed, often between the SMA and the aorta, leading to elevated pressure in the left renal vein. This can cause left flank pain and hematuria (from renal vein hypertension) and a left-sided varicocele (because the left testicular vein drains into the left renal vein; increased pressure causes dilation of the pampiniform plexus around the testis, a "bag of worms"). A kidney stone at the ureter would cause flank pain and hematuria, but not a varicocele. An AAA compressing the IVC could cause bilateral leg edema or affecting venous return, not specifically left renal vein issues with varicocele (and the IVC is on the right side). Thrombosis of left testicular vein itself could cause varicocele, but flank pain and hematuria suggest a renal vein issue. Left adrenal tumor is less common and would cause other issues (like hormone secretion) besides just vein compression; plus it might not explain varicocele unless it specifically blocked renal vein outflow.
69. A 68-year-old man with long-standing prostate cancer develops back pain. Imaging reveals metastatic lesions in the lumbar vertebrae. Prostate cancer commonly spreads to the vertebral column through which of the following routes?
(A) Prostatic venous plexus to the internal vertebral (Batson) plexus
(B) Thoracic duct lymphatic drainage
(C) Arterial embolization via prostatic artery
(D) Direct invasion through the bladder wall
(E) Inguinal lymph nodes to femoral bone marrow
Answer and Explanation: The correct answer is (A) Prostatic venous plexus to the internal vertebral (Batson) plexus. The prostate venous plexus (around the prostate) communicates with the internal vertebral venous plexus (Batson plexus) in the spine via valveless veins. This connection provides a route for prostate cancer cells to spread to the vertebrae, leading to bony metastases in the spine (common in prostate cancer). Lymphatic spread from prostate typically goes to internal iliac lymph nodes (not directly to thoracic duct until much later). Arterial spread of solid tumor is rare; metastasis is usually via veins or lymphatics. Direct invasion through the bladder would give local extension, not jump to bone. Inguinal nodes drain perineal skin and lower anus, not prostate (and not to bone). So the venous plexus route is the classic answer.
70. A 30-year-old man is undergoing surgery for a strangulated femoral hernia. The surgeon must open the femoral sheath to reach the hernia sac. Which of the following structures is NOT contained within the femoral sheath?
(A) Femoral nerve
(B) Femoral artery
(C) Femoral vein
(D) Deep inguinal lymphatics (in femoral canal)
(E) Femoral branch of genitofemoral nerve
Answer and Explanation: The correct answer is (A) Femoral nerve. The femoral sheath is an extension of the transversalis fascia that encloses the femoral vessels and the femoral canal (which contains lymphatics) in the upper thigh. It has three compartments for the femoral artery, femoral vein, and the femoral canal (which has lymph nodes/lymphatics, often called the canal of Cloquet). The femoral nerve, however, lies outside the femoral sheath, lateral to the femoral artery. It is not enclosed in the sheath. The femoral branch of the genitofemoral nerve does run in the sheath area (it usually runs with the femoral artery) and provides sensation to the upper anterior thigh. So, of the options, the femoral nerve is definitely not in the sheath, making it the correct answer to which is NOT contained.
71. A newborn is diagnosed with Treacher Collins syndrome, characterized by underdeveloped facial bones (especially the mandible) and malformed external ears. This results from aberrant neural crest development affecting the first pharyngeal (branchial) arch. Structures derived from the first branchial arch include which of the following?
(A) Maxilla, mandible, malleus and incus; muscles of mastication (CN V3)
(B) Stapes, styloid process; muscles of facial expression (CN VII)
(C) Greater horn of hyoid; stylopharyngeus muscle (CN IX)
(D) Thyroid cartilage; muscles of the larynx (CN X – recurrent laryngeal branch)
(E) Cricoid cartilage; intrinsic laryngeal muscles (CN X – recurrent laryngeal)
Answer and Explanation: The correct answer is (A) Maxilla, mandible, malleus and incus; muscles of mastication (CN V3). The first pharyngeal (branchial) arch gives rise to the maxillary and mandibular prominences (bones of the face like maxilla, zygoma, mandible), the Meckel’s cartilage derivatives (malleus and incus of the middle ear), and the muscles of mastication (masseter, temporalis, pterygoids), as well as mylohyoid, anterior belly of digastric, tensor tympani, tensor veli palatini. These are all innervated by the mandibular division of the trigeminal nerve (CN V3). In Treacher Collins, failure of neural crest to properly migrate into the first arch leads to hypoplasia of these structures. The second arch (option B) gives rise to the stapes, styloid process, lesser horn of hyoid, and muscles of facial expression (innervated by CN VII). The third arch (option C) gives the greater horn of hyoid and the stylopharyngeus muscle (innervated by CN IX). The fourth and sixth arches contribute to laryngeal cartilages (thyroid, cricoid, etc.) and laryngeal muscles (fourth arch by superior laryngeal of X, sixth by recurrent laryngeal of X).
72. An infant is born with DiGeorge syndrome (22q11 deletion), which includes thymic aplasia and hypocalcemia due to parathyroid aplasia. These defects result from failure of development of which embryologic structures?
(A) Third and fourth pharyngeal pouches
(B) First and second pharyngeal arches
(C) Second pharyngeal cleft
(D) Neural crest cells migrating to cardiac outflow tract
(E) Urogenital sinus
Answer and Explanation: The correct answer is (A) Third and fourth pharyngeal pouches. In DiGeorge syndrome, there is maldevelopment of the 3rd and 4th pharyngeal (branchial) pouches, which give rise to the thymus and parathyroid glands. Specifically, the third pharyngeal pouch forms the thymus and the inferior parathyroid glands, and the fourth pouch forms the superior parathyroid glands (and the ultimobranchial body that gives C-cells of thyroid). Failure of these pouches to develop properly leads to thymic aplasia (T-cell deficiency) and parathyroid aplasia (leading to hypocalcemia). The first and second arches are facial structures (not directly thymus/parathyroid). The second pharyngeal cleft normally obliterates; if it doesn’t, it can cause a branchial cleft cyst (not DiGeorge). Neural crest migration issues to the cardiac outflow cause conotruncal anomalies (which do occur in DiGeorge often, like Tetralogy of Fallot, truncus arteriosus), but the thymus/parathyroid specifically is pouches. Urogenital sinus forms bladder, urethra, etc., unrelated.
73. A 16-year-old girl has a painless, fluctuant mass on the lateral aspect of her neck, along the anterior border of the sternocleidomastoid muscle. It has been present since childhood and sometimes becomes infected. It does not move with swallowing. This is most likely a remnant of which embryologic structure?
(A) Second branchial (pharyngeal) cleft
(B) Thyroglossal duct
(C) First branchial cleft
(D) Third pharyngeal pouch
(E) Cervical sinus from first cleft
Answer and Explanation: The correct answer is (A) Second branchial (pharyngeal) cleft. A branchial cleft cyst classically presents as a lateral neck mass along the anterior border of the SCM, often appearing in late childhood or adolescence when it enlarges or gets infected. The most common is a persistent cervical sinus due to failure of the second pharyngeal cleft (and possibly 3rd and 4th) to obliterate. The second through fourth clefts normally form the cervical sinus, which should regress. If it persists, it can form a cystic mass in the lateral neck that does not move with swallowing (unlike a thyroglossal duct cyst, which is midline and moves with swallowing or tongue protrusion). A thyroglossal duct remnant would be midline (not lateral). First branchial cleft anomalies are often near the ear or under jaw (and can involve external auditory canal). Third pouch issues would be DiGeorge (thymus/parathyroid, not a neck cyst). Therefore, a lateral cyst near SCM is second cleft (branchial) origin.
74. During development of the heart, neural crest cells are involved in the formation of the aorticopulmonary septum. Failure of neural crest cells to migrate properly into the truncal and bulbar ridges of the developing heart can result in which congenital anomaly?
(A) Persistent truncus arteriosus
(B) Atrial septal defect (secundum type)
(C) Coarctation of the aorta
(D) Ventricular septal defect, membranous part
(E) Dextrocardia (Kartagener syndrome)
Answer and Explanation: The correct answer is (A) Persistent truncus arteriosus. Neural crest cells contribute to the conotruncal (aorticopulmonary) septum that divides the truncus arteriosus into the aorta and pulmonary trunk, as well as forming parts of the membranous interventricular septum. If neural crest migration fails, conotruncal abnormalities occur: these include persistent truncus arteriosus (failure of separation of aorta and pulmonary artery, leaving a single outflow vessel), transposition of the great arteries (if the septum doesn’t spiral), and Tetralogy of Fallot (anterior malalignment of septum). Membranous VSDs can also be due to neural crest issues because the membranous septum is formed when the conotruncal septum meets the muscular septum. However, persistent truncus is a direct example since it’s literally lack of septum formation. Atrial septal defect (secundum) is usually due to patent foramen ovale or septum secundum issues (not neural crest). Coarctation of aorta is related to ductal tissue, not neural crest directly. Dextrocardia in Kartagener’s is due to ciliary dyskinesia affecting organ rotation, not neural crest.
75. A 4-year-old boy has had recurrent ear infections and one was complicated by a perforation in the tympanic membrane. The tympanic cavity (middle ear) and auditory (Eustachian) tube are derived from which embryologic structure?
(A) First pharyngeal pouch
(B) First pharyngeal cleft
(C) Second pharyngeal pouch
(D) Second pharyngeal cleft
(E) Otic placode (surface ectoderm)
Answer and Explanation: The correct answer is (A) First pharyngeal pouch. The pharyngeal (branchial) pouches form on the endodermal side (inside) of the pharyngeal apparatus. The first pharyngeal pouch gives rise to the middle ear cavity (tympanic cavity) and the Eustachian (auditory) tube. The first pharyngeal cleft (external ectoderm) forms the external auditory canal (and the tympanic membrane is at the junction of first cleft and first pouch, with mesoderm in between). The second pouch forms the lining of palatine tonsils (tonsillar fossa). Second cleft normally is obliterated (if not, branchial cleft cyst). The otic placode is surface ectoderm that invaginates to form the inner ear (cochlea, vestibule, etc.), not the middle ear. So, middle ear and auditory tube = first pouch endoderm.
76. A 50-year-old man has a midline mass in the neck that moves upward when he protrudes his tongue. It is suspected to be a thyroglossal duct cyst. The thyroid gland originates from an outpouching of the embryonic pharynx at which site?
(A) Foramen cecum at the base of the tongue
(B) Second pharyngeal arch
(C) Fourth pharyngeal pouch
(D) Neural crest cells of branchial arches
(E) Floor of the primitive larynx
Answer and Explanation: The correct answer is (A) Foramen cecum at the base of the tongue. The thyroid gland begins development as an endodermal diverticulum in the midline of the pharynx, at the site of the foramen cecum on the dorsal surface of the developing tongue (between the anterior 2/3 and posterior 1/3 of the tongue). It then descends into the neck, passing anterior to the hyoid bone. The thyroglossal duct is the tract it follows; normally it disappears, but remnants can form a thyroglossal duct cyst anywhere along that path (often near hyoid). The second arch gives rise to structures like the styloid, facial muscles, etc., not the thyroid. The fourth pouch forms superior parathyroids and ultimobranchial body (C-cells). Neural crest cells contribute to C-cells of thyroid and maybe some connective tissue, but the thyroid gland tissue itself is from pharyngeal endoderm. "Floor of the primitive larynx" is not a precise term; the developing thyroid is from the pharynx floor at the foramen cecum region (which is the answer given by A).
77. A 46,XX newborn is found to have a bicornuate uterus (uterus with two horns) as seen on imaging, which can lead to reproductive complications. A bicornuate uterus results from incomplete fusion of which embryologic structures?
(A) Paramesonephric (Müllerian) ducts
(B) Mesonephric (Wolffian) ducts
(C) Urogenital folds
(D) Genital tubercle
(E) Labioscrotal swellings
Answer and Explanation: The correct answer is (A) Paramesonephric (Müllerian) ducts. In females, the paramesonephric ducts (Müllerian ducts) develop into the fallopian tubes, uterus, cervix, and upper vagina. A bicornuate uterus occurs when there is incomplete or failed fusion of the two paramesonephric ducts. This leads to a uterus with two “horns” and a common cervix or two cervices, depending on severity. This can cause infertility or miscarriage issues. Mesonephric (Wolffian) ducts mostly regress in females (they form male structures if male). Urogenital folds become labia minora in females or spongy urethra in males. Genital tubercle becomes clitoris or penis glans. Labioscrotal swellings become labia majora or scrotum. None of those are about forming the uterus shape.
78. The round ligament of the uterus travels through the inguinal canal and attaches to the labia majora. It is the female remnant of which embryologic structure?
(A) Gubernaculum
(B) Processus vaginalis
(C) Mesonephric duct
(D) Paramesonephric duct
(E) Urachus
Answer and Explanation: The correct answer is (A) Gubernaculum. The gubernaculum in the embryo is a fibrous cord that aids in the descent of the gonads. In males, it helps pull the testes into the scrotum and then remnants anchor the testis (gubernaculum becomes the scrotal ligament). In females, the gubernaculum attaches to the developing uterus and ovary, and it becomes the ovarian ligament (between ovary and uterus) and the round ligament of the uterus (from uterus through inguinal canal to labia majora). The processus vaginalis is a peritoneal extension that in males becomes the tunica vaginalis (if patent leads to hydrocele); in females it usually obliterates (canal of Nuck if persistent). Mesonephric duct gives male internal structures (vas deferens, etc.) and in females largely regresses. Paramesonephric duct forms female internal structures (uterus, etc.), not the round ligament. Urachus is from allantois, connecting bladder to umbilicus (median umbilical ligament).
79. A 18-year-old phenotypic male (appearing male) is found to have both male and female internal reproductive structures: he has a uterus and fallopian tubes as well as vas deferens. He is diagnosed with persistent Müllerian duct syndrome due to a deficiency of Müllerian-inhibiting factor (Anti-Müllerian hormone) during development. Which cells normally secrete the substance responsible for preventing development of a uterus in males?
(A) Sertoli cells of the testes
(B) Leydig cells of the testes
(C) Theca cells of the ovary
(D) Granulosa cells of the ovary
(E) Syncytiotrophoblast cells of placenta
Answer and Explanation: The correct answer is (A) Sertoli cells of the testes. In a male (46,XY) fetus, the Sertoli cells of the testes produce Müllerian-inhibiting factor (MIF), also known as Anti-Müllerian Hormone (AMH), which causes the regression of the paramesonephric (Müllerian) ducts. If Sertoli cells do not produce MIF (or if there's insensitivity to it), the Müllerian ducts will persist and develop into female internal structures (uterus, fallopian tubes, etc.) even in a genetic male. At the same time, the Leydig cells produce testosterone to develop the mesonephric ducts into male internal structures. Leydig cells secrete testosterone (and DHT via conversion), but MIF specifically comes from Sertoli cells. Theca and granulosa are ovarian cells (involved in estrogen production, no role in male duct regression). Syncytiotrophoblast secretes hCG, etc., not relevant to internal duct differentiation.
80. A 1-week-old male infant is brought in with continuous dribbling of urine from his umbilicus. He is also noted to have irritation around the umbilical area. This is most likely due to a patent connection between the bladder and the yolk sac known as:
(A) Urachal fistula (patent urachus)
(B) Meckel diverticulum
(C) Omphalocele
(D) Vitelline fistula
(E) Patent processus vaginalis
Answer and Explanation: The correct answer is (A) Urachal fistula (patent urachus). The urachus is a remnant of the allantois, which in development connects the bladder to the yolk sac (through the umbilicus). Normally, it obliterates to become the median umbilical ligament. If it remains patent, there can be a direct connection from the bladder to the umbilicus, resulting in urine discharge from the umbilicus (urachal fistula). This infant’s symptoms of urine leaking from the navel indicate a patent urachus. A Meckel diverticulum is a vitelline duct remnant in the ileum (would cause bowel-related issues, possibly meconium from umbilicus if it were a fistula, but here it’s urine). Omphalocele is a midline defect with abdominal contents in a sac at birth (not the issue here). A vitelline fistula (patent vitelline duct) would connect ileum to umbilicus, causing feculent discharge from the umbilicus. Patent processus vaginalis leads to hydrocele or indirect hernia in the inguinal region, not umbilical urine.
81. A 3-year-old boy presents with a swelling in his right scrotum. The swelling increases in size when he is active during the day and seems to diminish overnight when he is lying down. It is translucent on transillumination. Which embryologic abnormality is the most likely cause?
(A) Patent processus vaginalis leading to a communicating hydrocele
(B) Failure of closure of the umbilical ring leading to an umbilical hernia
(C) Patent urachus leading to a urachal cyst
(D) Incomplete fusion of urethral folds leading to hypospadias
(E) Excess androgen exposure in utero leading to labioscrotal fusion
Answer and Explanation: The correct answer is (A) Patent processus vaginalis leading to a communicating hydrocele. The scenario describes a hydrocele in the scrotum (fluid-filled swelling that transilluminates). A communicating hydrocele occurs when the processus vaginalis (an extension of peritoneum into the scrotum during testicular descent) remains patent, allowing peritoneal fluid to pass into the scrotum. It often enlarges during the day (when upright, more fluid drains in) and reduces when supine (fluid drains back). It's essentially like an indirect inguinal hernia sac but only fluid (no bowel). Umbilical ring closure failure is umbilical hernia (bowel at umbilicus). Patent urachus causes urine leakage or cyst along urachus tract, not scrotal swelling. Hypospadias is misplacement of urethral opening due to urethral folds not fusing completely (nothing to do with scrotal hydrocele). Excess androgen would virilize female or cause early male dev issues, not a hydrocele. So patent processus vaginalis is correct.
82. A 35-year-old pregnant woman is informed that her fetus has a congenital diaphragmatic hernia due to a developmental defect. The hernia is on the left posterolateral side and has allowed abdominal organs to herniate into the chest, affecting lung development. This defect is most likely caused by failure of which embryologic structure to develop or fuse properly?
(A) Pleuroperitoneal membrane
(B) Septum transversum
(C) Lateral body wall folds
(D) Dorsal mesentery of the esophagus
(E) Pleuropericardial membrane
Answer and Explanation: The correct answer is (A) Pleuroperitoneal membrane. The most common form of congenital diaphragmatic hernia is a Bochdalek hernia, which is a defect in the posterolateral diaphragm, usually on the left side. This results from failure of the pleuroperitoneal membrane to close the pericardioperitoneal canal during development. Normally, the diaphragm forms from several components: septum transversum (central tendon), pleuroperitoneal membranes, dorsal mesentery of esophagus (forms crura), and muscular ingrowth from lateral body walls. A posterolateral defect is typically due to the pleuroperitoneal membrane not fusing with the rest. The septum transversum mainly forms the central tendon and anterior portion; a defect there is rarer (central hernia). The pleuropericardial membrane separates pericardial from pleural cavities (heart from lungs), not the diaphragm. So pleuroperitoneal membrane defect is correct.
83. A newborn chokes and coughs during its first feeding and has excessive drooling. A nasogastric tube cannot be passed into the stomach. This presentation is most consistent with which of the following anomalies?
(A) Tracheoesophageal fistula with esophageal atresia
(B) Isolated cleft palate
(C) Choanal atresia (blockage of posterior nasal passages)
(D) Hypertrophic pyloric stenosis
(E) Laryngeal atresia
Answer and Explanation: The correct answer is (A) Tracheoesophageal fistula with esophageal atresia. The scenario is classic for the most common type of tracheoesophageal fistula (TEF), where the proximal esophagus ends in a blind pouch (atresia) and the distal esophagus is connected to the trachea. Newborns with this have drooling (because saliva can’t pass to stomach), choke and cough with feeds as the milk goes into the blind pouch and overflows potentially into the trachea. They often have immediate regurgitation of feeds and can't pass an NG tube down (it will stop in the blind pouch). This is a congenital defect of the tracheoesophageal septation. Cleft palate causes feeding difficulties but more with nasal regurgitation, not inability to pass NG tube. Choanal atresia causes difficulty breathing (especially at rest if bilateral, because babies are nose breathers) and cyanosis relieved by crying, not so much choking on feeding. Pyloric stenosis presents at ~3 weeks old with projectile vomiting, not at first feed. Laryngeal atresia is rare and would cause immediate airway blockage (would not allow crying or breathing at all).
84. A newborn is diagnosed with persistent truncus arteriosus, where a single arterial trunk comes out of the heart supplying both systemic and pulmonary circulations (accompanied by a ventricular septal defect). This condition is due to the failure of which of the following embryologic processes?
(A) Neural crest cell migration to form the aorticopulmonary septum
(B) Septum primum and septum secundum fusion
(C) Endocardial cushion formation in the AV canal
(D) Apoptosis in the interventricular septum
(E) Remodeling of the vitelline arteries
Answer and Explanation: The correct answer is (A) Neural crest cell migration to form the aorticopulmonary septum. Persistent truncus arteriosus occurs when the truncus arteriosus fails to divide into the aorta and pulmonary trunk. Normally, neural crest cells contribute to the formation of the aorticopulmonary septum that splits the truncus arteriosus and also fuses with the interventricular septum. Failure of these processes (often due to neural crest problems) leads to a single outflow vessel (truncus) and usually a VSD because the membranous septum (also crest-derived) didn't form properly. Septum primum/secundum fusion issues cause atrial septal defects (like patent foramen ovale). Endocardial cushion defects cause things like AV septal defects (common in Down syndrome) and contribute to both atrial and ventricular septa. Apoptosis in interventricular septum isn’t a described mechanism; the muscular septum forms by growth, not apoptosis. Remodeling of vitelline arteries pertains to GI arterial supply (celiac, SMA, IMA development), not truncus separation.
85. A 25-year-old man is stabbed in the back. The knife severs the ligamentum flavum between two lumbar vertebrae. Which structure, related to the vertebral column, was most directly penetrated?
(A) The ligament connecting the laminae of adjacent vertebrae
(B) The ligament connecting the anterior surfaces of vertebral bodies
(C) The ligament connecting the tips of the spinous processes
(D) The ligament connecting the transverse processes
(E) The ligament separating the dura mater from the vertebral canal
Answer and Explanation: The correct answer is (A) The ligament connecting the laminae of adjacent vertebrae. The ligamentum flava (plural: ligamenta flava) are paired ligaments (with a yellowish elastic tissue) that connect the laminae of adjacent vertebrae on the posterior wall of Answer and Explanation (continued): ...the spinal canal. Penetration of the ligamentum flavum would occur if the knife went between vertebrae through the back (it is also the structure that is "popped" through during an epidural or lumbar puncture). The anterior longitudinal ligament runs along the front of vertebral bodies (not likely hit from a posterior approach). The supraspinous ligament connects the tips of spinous processes in a continuous cord along the back; a knife between vertebrae would more likely hit interspinous or ligamentum flavum rather than the supraspinous (unless it skimmed the surface). The intertransverse ligaments connect transverse processes, far lateral. There is no single "ligament separating dura from canal"—the dura is separated by the epidural space, which contains fat and the internal vertebral venous plexus, not a distinct ligament. Therefore, the ligamentum flavum, connecting adjacent laminae, was most directly penetrated.
No Comments Were Found
You need to login to comment